Categories
Exam Questions Harvard Suggested Reading Syllabus

Harvard. Economics of population growth. Syllabus and examination. Kuznets, 1963

Simon Kuznets was born in Pinsk (Russian Empire, now Belarus) April 30, 1901. He went to secondary school in Rovno and in Kharkov. In 1918–1921 he attended Kharkov Commercial Institute, after which he worked 1921–1922 in the Department of Labor Statistics of South Bureau of Council of Trade Unions. His family emigrated to the United States in 1922.

The Kharkiv National University of Economics in Ukraine was named after Simon Kuznets in 2013.

Previously transcribed and posted here at Economics in the Rear-view Mirror: materials from Simon Kuznets’ 1960-61 Harvard course, Economics 203 “Economic Growth and Comparative Economic Structures”.

____________________________

Course Announcement

Economics 286. Economics of Population Growth
Half course (spring term). Tu., 2-3:30. Professor Kuznets.

A review of long-term trends in growth and structure of population and of their economic implications.

Source: Harvard University. Faculty of Arts and Sciences. Courses of Instruction for Harvard and Radcliffe, 1962-1963. Official Register of Harvard University, Vol. LIX, No. 17 (August 20, 1962), p. 108.

____________________________

HARVARD UNIVERSITY
Department of Economics
Economics 286
Professor Kuznets

Spring Term, 1963

List of Readings (preliminary)

The Malthusian Theory

  1. T. R. Malthus, An Essay on the Principle of Population, 1st edition 1798, reprinted by Macmillan 1929 (or other reprints)
  2. James Bonar, Malthus and His Work, London 1885, reprinted in 1924, (particularly Book I, pp. 1-207)

(For Browsing)

  1. Kenneth Smith, The Malthusian Controversy, London 1951
  2. Harold A. Boner, Hungry Generations, New York 1955

The Low-level Equilibrium Trap

  1. R. R. Nelson, A Theory of the Low-Level Equilibrium Trap, American Economic Review, December 1956, pp. 894-908
  2. Harvey Leibenstein, Economic Backwardness and Economic Growth, New York 1957, Chapter 10, pp. 117-173

Historical Background

  1. United Nations, The Determinants and Consequences of Population Trends, New York 1953, Chapter II, pp. 5-20

Mortality

  1. United Nations, volume listed under (7), Chapter IV, pp. 47-70
  2. George J. Stolnitz, A Century of International Mortality Trends, Population Studies, vol, IX, no. 1, July 1955, pp. 24-55 and vol. X no. 1, July 1956, pp. 17-42

(For Browsing)

  1. Thomas McKeown and R. S. Record, two papers on causes of decline in mortality in England in the 18th and in the 19th centuries, Population Studies, vol. IX, 1955, pp. 119-41 and vol. XVI, 1962, pp. 94-122

Fertility

  1. United Nations, volume listed under (7), Chapter V, pp. 71-97
  2. Frank Lorimer, Culture and Human Fertility, UNESCO 1954, pp. 15-251
  3. Kingsley Davis and Judith Blake, Social Structure and Fertility: An Analytic Framework, Economic Development and Cultural Change, vol. IV, no. 3, April 1956, pp. 211-35
  4. Gary S. Becker, An Economic Analysis of Fertility, Demographic and Economic Change in Developed Countries, Universities-NBER Committee volume, Princeton 1960, pp. 209-240
  5. E. E. Hagen, Population and Economic Growth, American Economic Review June 1959, pp. 310-327

Migration

  1. United Nations, volume listed under (7), Chapter VI, pp. 98-134

Economic Implications

  1. United Nations, volume listed under (7), Chapter XIII, pp. 220-38
  2. Simon Kuznets, Population Change and Aggregate Output, in the volume listed under (6), pp. 324-351
  3. Ansley J. Coale and Edgar M. Hoover, Population Growth and Economic Development in Low Income Countries, Princeton 1958, Part Five, pp. 295-335

Source: Harvard University Archives. Syllabi, course outlines and reading lists in Economics, 1895-2003, Folder “Economics, 1962-1963 (2 of 2)”.

____________________________

HARVARD UNIVERSITY
Economics 286
Examination, May 23, 1963

Please answer six and no more than eight questions, choosing at least one question in each of the four Roman numeral groups.

A detailed outline of an answer is a good substitute for writing out the answer in full,

Please write legibly.

I

  1. What are the connections in the Malthusian theory of population growth among the rate of natural increase of population, the “Iron” law of wages, and the law of diminishing returns? In answering consider birth and death rates separately.
  2. What assumptions are made in the low-level equilibrium trap theories concerning the connection between death rates and per capita income? Concerning the connection between birth rates and per capita income? Discuss the validity of these assumptions.

II

  1. What factors made for significant declines in mortality in the period since the late 18th century? In outlining the groups of factors involved, suggest why the declines were delayed for long periods in the 19th century in many developed countries.
  2. Indicate the differences in magnitudes of decline in mortality among (a) age groups; (b) sex groups; (c) urban and rural population. Select any two of these for a discussion of the factors that may have been responsible for the differences in magnitude of mortality decline.

III

  1. Outline the major trends in the birth rates since late 18th century in: (a) older developed countries of Europe; (b) younger offshoots of Europe overseas; (c) underdeveloped countries. In answering the questions, indicate, in case trends are significant, whether they apply to crude rates alone or also to rates per woman of child-bearing age and per married woman of child-bearing age.
  2. What groups of factors have been suggested to account for the downward trends in birth rates in the developed countries? In outlining these, and the relevant theories, mention also the cross-section differentials in birth rates that provide the empirical base for such theories.
  3. What do the current differences in crude birth rates between developed and underdeveloped countries reflect in the way of different marriage rates, different ages of women at marriage, differential fertility by age of mother and age of father, total number of children in a completed family? Suggest some economic implications of these differences.

IV

  1. What relationship between population trends and economic trend account for much of internal and external migration observed since the late 18th century in and among developed countries? In answering, try to specify as closely as possible the economic and the population trends involved.
  2. What effects on the efficiency of labor can be ascribed to internal (and external) migration in developed countries? Indicate briefly the various types of effect that might be usefully distinguished.
  3. What effects on the pattern of life and consumption of the population can be ascribed to internal (and external) migration in developed countries? Indicate briefly the various type of effect that might be usefully distinguished.

Source: Harvard University. Faculty of Arts and Sciences. Papers Printed for Final Examinations [in] History, History of Religions, … , Economics, …Naval Science, Air Science. June 1963. In bound volume: Social Sciences, Final Examinations, June 1963 (HUC 7000.28, Vol. 147 of 284).

Image Source: Simon Kuznets portrait from 1971 in Wikipedia Commons.

Categories
Economics Programs Harvard Undergraduate

Harvard. President of Harvard responding to Economics Dept Visiting Committee Report, 1952

In can hardly be surprising that the relationship between a visiting committe dominated by business people and an academic department of economics might suffer from incompatible visions of what constitutes “good” economic research, teaching and policy.

The Harvard’s visiting committee in 1950 thought the secret sauce missing from a proper economics department was a professorial advocate of business enterprise to counterbalance an alleged dominance of Keynesian and socialist positions. This was the principal criticism of the committee. Other shortcomings claimed were inadequate planning/coordination between graduate and undergraduate programs, too few professorial heavyweights teaching in the undergraduate program, and a tendency for professors’ policy consulting activities to crowd out their expected instructional and research duties.

The chairman of the economics department’s visiting committee at mid-century was the Chicago businessman, Clarence B. Randall (Harvard A.B., 1912).

Harvard President’s James B. Conant’s conclusion in his 1952 response:

Over the last fifteen years the Department of Economics has been at fault in not attempting to meet the Visiting Committee in a spirit of wholehearted cooperation. The Board of Overseers has been at fault, I venture to suggest, by not widening the membership of the Visiting Committee to include more professional economists and more businessmen who have been working closely with university economists.

___________________________

For private circulation NOT for publication

CONFIDENTIAL REPORT OF THE PRESIDENT OF THE UNIVERSITY TO THE TWO GOVERNING BOARDS ON THE DEPARTMENT OF ECONOMICS OF THE FACULTY OF ARTS AND SCIENCES

(Accepted by President and Fellows of Harvard College on January 7, 1952, and by the Board of Overseers on January 14, 1952)

TO THE PRESIDENT AND FELLOWS OF HARVARD COLLEGE:
TO THE BOARD OF OVERSEERS OF HARVARD COLLEGE:

On November 27, 1950, the Chairman of the Committee to Visit the Department of Economics reported in writing to the Board of Overseers. The report, which is attached, raises serious questions about future appointments to the permanent staff. As the President of Harvard is responsible for presenting to the two Governing Boards the names of those who he is persuaded should be the future professors, such questions concern him directly. I have, therefore, felt obliged to examine personally the validity of the “most pressing criticism” in the report of November 27, 1950. My findings and recommendations are contained in this confidential report which I hope may be accepted by formal vote of each of the Governing Boards in January.

For a number of months now I have been studying the teaching of economics at the university level. In so doing, I have talked with academic economists on three continents, with those employed by business and by government, and with members of the business community. I am convinced that the Harvard Department of Economics is a distinguished department. As far as the types of economic theory and analysis presented to the students are concerned, it is typical of departments of economics in the leading universities of the English-speaking world. The educational problems discussed in the first seven paragraphs of the Visiting Committee’s report are likewise typical. Indeed, they are not confined to teachers of economics. Difficulties in reconciling the needs of the undergraduate and the graduate student with the scholarly pursuits of a professor and calls for expert services are to be found in the majority of the departments of the Faculty of Arts and Sciences. It is the constant aim of the administration to hold the balance even between the various types of teaching and research. To this end, the informed criticisms of visiting committees are helpful. But important as these questions are, they do not warrant a special written report from the President of the University. Therefore, I shall state here only that I am satisfied that the department is taking steps to improve the teaching of undergraduates and will take further steps in this direction, particularly as regards the introductory course.

My examination of the status of economics in American universities today has revealed the fact that in at least fourteen major universities questions are being raised by persons who are not economists about the teaching of economics. It is a curious fact that at the same period of history in which there is a certain degree of national unrest about academic economists, one group of businessmen (the Committee for Economic Development) is closely associated with professors of economics in a series of investigations of vital problems. It seems a pity that the confidence that part of the business community has in at least some university economists does not receive as much publicity as do the attacks by others who claim our schools and colleges are teaching “collectivism.” Not that any such charge is made by the Overseers Committee here at Harvard. What is criticized is only “that the Department as presently constituted lacks balance with respect to the viewpoint of its members.” This is a reasonable criticism and warrants a careful investigation. The Visiting Committees of the Board of Overseers are both special pleaders for and critics of the departments or faculties which they visit. That doubts and questions should be raised by them from time to time on any or all matters is obviously of great advantage to the University.

At the outset of my inquiry the difficulties of formulating criteria for cataloguing the viewpoints of economists became evident. I tried the test of Keynesian and anti-Keynesian but soon discovered I was using a totally inadequate analytic tool; I became convinced that Keynes himself was an anti-Keynesian before he died. The Overseers report states categorically that there are “one or more socialists” in the Department. With this statement I must respectfully but firmly disagree and in so doing point out both the difficulties and the necessity of defining terms in the social sciences. The term “socialist” as used in countries where socialism is a live political issue means one who advocates by democratic political action “the nationalization of the means of exchange, production and distribution.” It might be a good thing to have a socialist on the staff of a department of economics, but as a matter of fact there are no socialist professors of economics at Harvard today.

One could classify economists, at least theoretically, in terms of their political beliefs, but except for communists and socialists this is a very difficult matter in the present flux of political opinion. Furthermore, people’s political convictions, like their religious beliefs, are often subject to violent change. Everyone speaks of the dangers of introducing political criteria into the consideration of academic appointments. If analyzed, I believe these dangers stem largely from the fact that political views do not represent a bias relevant to an academic intellectual discipline as does a philosopher’s adherence to a philosophic doctrine such as idealism or logical empiricism. Political opinions are temporary, emotional, and subject to change under social duress; it is to avoid such duress that politics and religion are considered “out of bounds” in judging persons for academic posts in the United States in the mid-twentieth century.

The Chairman of the Visiting Committee in his report speaks of a “social spectrum.” I have attempted to use this concept to classify present-day economists as radical or conservative without getting into the political quagmire to which I have just referred. I have had little success except that in a vague sort of way a number of informed observers have expressed the view that the leading universities of the nation were about equally radical or conservative as regards their departments of economics. But if the President is to direct a department or an ad hoc committee as to future appointments, he must have some more definite criteria as to a man’s position in the social spectrum, and these I have failed to find. For example, I find it difficult to decide whether advocacy of strengthening the Sherman Anti-Trust Act is radical or conservative. I ask myself was the Harvard Department a generation ago radical or conservative? In retrospect it seems conservative to many; forty years ago it was considered radical, as the free-trade point of view predominated. When I first took office, some discussion in the Board of Overseers indicated that there were those who used a man’s attitude towards organized labor as the touchstone of his radical or conservative outlook. This is no longer so. As a consultant to the Government, an economist may take a strong position as to need for immediate drastic action to offset a depression or control an inflation. In recent years such rather technical economic opinions have bulked large in some people’s minds in classifying economists as being to the left or right. For example, if you confine your attention to fiscal policy in the immediate past, you could find two professors in the Harvard Department today to place in opposition to one another. But I have become convinced that no criteria of lasting value in terms of a social spectrum can be devised for the guidance of any body charged with responsibility for nominating candidates for appointment in a department of economies.

Balance in a department of economics today, I have concluded. should be first, balance between special fields, and second, balance between types of methods employed by the professors. As to fields such as labor, agriculture, money and banking, the Committee has raised no issue; there is no problem special to economics here. The question arises in chemistry, in history, in biology, to name but three instances. As far as I can, I insist that for the permanent appointments a balance of fields be a secondary consideration since an adequate coverage of all subjects can be taken care of through appointing assistant professors. Rigid insistence on having each field represented by a permanent appointment limits the number of candidates and tends to encourage the appointment of “good” rather than “excellent” men. The same is true as regards methods. Yet, as in the case of special fields, I must admit that there should be some effort made to achieve a balance among the permanent members of the staff, provided that in so doing there is no sacrifice of the quality of the appointments.

From my studies I have concluded that a layman may well classify economists in three groups according to the methods they employ: (1) theorists using models and the logical deductive approach; (2) investigators concerned with statistical aggregate analysis; (3) an empirical approach to specific problems as illustrated by the ad hoc case study of business problems. I have the impression that, in general, college departments of economies are relatively weak as regards the third of these methodological classes. In contrast, the Harvard School of Business Administration is strong here and until recently has been less concerned with the other two methodological approaches. The Harvard Department of Economies, if I understand the Chairman correctly, has felt for some time that this relative methodological weakness needed correction. Two professors of the Business School faculty are now giving a course in the Department. Further, in a letter replying to the criticism of the Visiting Committee, the Chairman, speaking for the Department, writes:

“As a result of your letter and our discussions with you, we have carefully considered the question of balance of fields of interest in the Department. While we are not prepared to concede that we are more unbalanced than other departments of economics, we agree that our balance could be improved. In particular, our Department, like most others, could be improved if we had at least one member whose major interest was what we might call the economics of enterprise. We believe that this is a field of growing importance, but it is one that has not been widely cultivated in economics departments. An additional member of the Department who could give an undergraduate course in the Economics of Enterprise and a graduate seminar on the same lines would contribute to a better balance of the Department. We suggest that the Corporation consider allocating an additional permanent position to the Department at the full professor level.”

To follow this suggestion would lead to no end of difficulties in the Faculty of Arts and Sciences; other departments would be quick to press for an increase in their quota of permanent places. But I am glad to report that much the same end can be accomplished because the Dean of the Business School has expressed his interest in a joint appointment. With his consent and with the concurrence of the Provost I therefore recommend that the Corporation agree to appoint one full professor of economics over and above the quota allowed by the schedule of appointments for the Faculty of Arts and Sciences established a decade or so ago. I further recommend that this professor hold an appointment in three faculties, namely, the Faculty of Arts and Sciences, the School of Business Administration, and the School of Public Administration, and that his salary be charged to the three faculties in such amounts as the President shall determine. Further, that the nomination for the new chair be made by the permanent members of the Department of Economics of the Faculty of Arts and Sciences and six members of the Faculty of the School of Business Administration appointed by the President after consultation with the Dean, the two groups to sit together as a nominating committee, and the name or names thus nominated to be passed on by an ad hoccommittee as is usual in the Faculty of Arts and Sciences.

The directive to the nominating committee would be as follows: to submit one or more names of men of character, high scholarly distinction and first-rate teaching ability who have an understanding of business as it is actually operated. To that end, the man in question should have had contact as a scholarly investigator or consultant with the operations of industry and commerce; he should have an awareness of the positive role of business enterprise in a changing and developing economy. His teaching would be directed towards presenting to Harvard College students a realistic view of business management and its relation to the total economy. If this report is accepted by the two Governing Boards, I shall proceed with this appointment.

The last paragraph of the report of the Visiting Committee requires special comment. It is stated that “This problem of balance within the Department will not be solved by the ad hoc committees. There only the qualifications of the particular man are under consideration. It is not the function of such a committee to determine whether the man’s appointment will restore balance or add to lack of balance.” I must beg leave to take exception to this exposition of the role of the ad hoc committees, and in so doing call the attention of the new members of the Board of Overseers to the Report of the Special Committee to Review the Operation of the “Ad Hoc” Committees in the Faculty of Arts and Sciences. The ad hoc committees determine nothing definitely, that is true. But they advise the President and through him the two Governing Boards as to whether or not the appointment suggested by the department is the best possible appointment that can be made all things considered; and among the considerations are the needs of the department for teachers and scholars in this or that subdivision of the field and with this or that scholarly technique at their disposal.

An ad hoc committee does far more than pass on “the qualifications of the particular man under consideration”; an ad hoc committee often recommends that someone other than the candidate nominated by the department should be considered. And such recommendations have more than once resulted in the appointment of a person who had not even been on the list considered by the department. As presiding officer of these ad hoc committees, I can certify from experience as to their effectiveness; I can assure the members of the two Governing Boards that in the field of economies, as elsewhere, I shall endeavor to see to it that the names I present are in my opinion the names of the best people to appoint. For the temporary appointments at the assistant professor and instructor level, the Dean of the Faculty performs the same function as the ad hoc committee.

The acceptance of this report by the two Governing Boards will mean that they agree with me that the issue of an individual’s radicalism or conservatism or a man’s political attitude is inadmissible in connection with his appointment. (I have made it clear elsewhere that I would not be a party to the appointment of a member of the Communist Party, for reasons I need not here repeat.) Balance between special fields and different methodological approaches in economics we shall strive for, and I recognize that it is a proper function of the Governing Boards from time to time to see that this is done, though not with respect to a particular appointment. There will be no directives to the nominating group or the ad hoc committee in terms of a man’s political views or his position on what has been referred to as a social spectrum. Since that will be the case in economics as in other fields, only the validity of the evidence I present as to a man’s character and competence as a scholar and teacher will be relevant to the decision about an individual in either the Corporation or the Board of Overseers. Once the ground rules are determined by the two Boards, the responsible officials must be trusted to operate within them. On no other basis, in my opinion, can this University function satisfactorily.

In conclusion I wish to express my deep appreciation for the spirit in which the report of the Visiting Committee is written. The Chairman states that it is not his intention “to initiate controversy or to suggest that we view with extreme alarm any phase of the Department’s work.” And later in the report he states, “No friend of academic freedom need fear the purpose which underlies our comment on this matter. . . We would be the first to insist that a professor must teach that which he honestly believes and we know that the fact that this differs from viewpoints which we may hold as individuals is altogether immaterial.”

I feel sure that the Chairman speaks not only for his Committee but for the whole Board of Overseers when he makes these statements, which are by no means universally accepted today in the United States. I need hardly state that even questions concerning the criteria to be employed in judging candidates for appointment in controversial areas — questions that touch sensitive nerve centers in a university today — are quite within the province of the visiting committees. Indeed, no one can have the slightest objection to the critical discussion of these matters in a university, provided, as in this case, the discussion is initiated by duly constituted committees within a constitutional framework.

Unfortunately, the public criticisms of professors in these days do not all conform to the restrained pattern set by this report. Rather the demands for “firing” or “muzzling” professors or censoring textbooks have increased in number and intensity in the last few years. I suppose all members of the two Boards are familiar with such irresponsible attacks as those of Zoll in his “Reducators” and the rather violent statements about the teaching of economics emanating from more reputable sources. I mention these matters for they have a certain relation to the problem that a president of a university faces today when he must recommend action in a controversial area such as economics. The analogy with his distant predecessors’ problems in theology comes to mind.

The existence of hostile critics and extremists makes it imperative for fair-minded men concerned with the future of education to thrash out their differences of opinion around a table. Over the last fifteen years the Department of Economics has been at fault in not attempting to meet the Visiting Committee in a spirit of wholehearted cooperation. The Board of Overseers has been at fault, I venture to suggest, by not widening the membership of the Visiting Committee to include more professional economists and more businessmen who have been working closely with university economists. But the situation is better in both respects than it was a few years past; in my opinion it can be still further improved.

In these critical days when economic decisions play so vital a part in determining national and international policies, it is unfortunate that an atmosphere of hostility exists to some degree throughout the country between the management of industry and academic economists. Whatever can be done here at Harvard to increase the understanding between men of good will within and without the University cannot fail to be of service to the nation.

Respectfully submitted,
JAMES B. CONANT

January 3, 1952

___________________________

No. 2 REPORT OF THE COMMITTEE TO VISIT THE DEPARTMENT OF ECONOMICS

TO THE BOARD OF OVERSEERS OF HARVARD COLLEGE:

Some three years have elapsed since a written report has been submitted to the Board of Overseers on behalf of the Committee to Visit the Department of Economics.

Once each year since that last report the Committee has met socially at dinner with the members of the Department, has met privately with the Provost to discuss the problems of the Department, and then has met in executive session. In addition to these annual meetings, the individual members of the Committee have endeavored conscientiously to inform themselves privately about the organization of the Department and the program of instruction.

It is not our purpose in making a report at this time to initiate controversy or to suggest that we view with extreme alarm any phase of the Department’s work. We do have apprehensions, but our viewpoints have been fully expressed to the President and the Provost, both of whom have encouraged us at all times to be frank in such criticisms as we have had to present.

The particular points which we have to make may be rather briefly stated.

It will be recalled that in the last previous report attention was drawn to the fact that the Department seems to lack over-all planning. We still think that a sound criticism. The Department contains brilliant individuals who are passionately devoted to their particular approach to the subject matter. But it could hardly be fairly said that their efforts are coordinated into a comprehensive plan, whether it be furnishing undergraduates a well-rounded training in economies or carrying on research at the graduate level.

Another criticism which has been made by others is that the members of the Department seem to emphasize the importance of their work with graduate students to the prejudice of undergraduate instruction. Few, if any, of the distinguished members of the Department are ever seen or heard by undergraduates, and we think this a great loss. We believe it to be important that Harvard give her best to those undergraduates who for the first time in their lives are approaching this highly significant subject, in order that their minds may be stimulated and broadened by the inspiration of great teaching.

Another criticism which has been made is that too many members of the Department absent themselves for extensive periods from their University duties. We recognize the demands that Government properly makes upon the University for the loan of Harvard economists. We also believe that a Harvard professor can benefit through working on projects for business managements or labor organizations. There must, however, be reasonable balance between such occasional outside employment and the first duty of the professor to the University. We believe there is ground for this criticism of the Economics Department and that the matter warrants careful study by the Provost and the Department Chairman.

The most pressing criticism, however, which we have to offer is that the Department as presently constituted lacks balance with respect to the viewpoints of its members. It is particularly the trend toward lack of balance which disturbs us.

No friend of academic freedom need fear the purpose which underlies our comment on this matter. We would be the first to insist that a professor must teach that which he honestly believes and we know that the fact that this differs from viewpoints which we may hold as individuals is altogether immaterial. This is too obviously right to need discussion.

But that is a totally different question from that of believing that all viewpoints should be ably represented within the Department. The most determined champion of academic freedom would join us we believe in urging the importance of balance in a controversial field. This is an old and familiar problem at universities; for example, in the departments of philosophy. There we believe that sound university administration always seeks such balance. We suggest that comparable balance is not presently to be found within the Department.

We have in the Department, for example, one or more Socialists, some zealous followers of British economist, John Maynard Keynes, and some who advocate the extension of economic controls by Government. Some of these men are nationally known for their views and are both active and zealous in promoting Them. But on the other side of the social spectrum, the Department seems to lack men of equal ability and zeal who hold opposing views and are prepared to teach them.

This problem of balance within the Department will not be solved by the ad hoc committees. There only the qualifications of the particular man are under consideration. It is not the function of such a committee to determine whether the man’s appointment will restore balance or add to the lack of balance. That delicate question can be solved only through leadership by the President, the Provost, and the Chairman. We have confidence that they share our concern and we hope that this statement of our apprehensions will be helpful to them.

CLARENCE B. RANDALL

November 27, 1950

Source: Harvard University Archives. Confidential Report of the President of the University to the Two Governing Boards of the Department of Economics of The Faculty of Arts and Sciences, 1952 January 3 (Archives Stacks UAI.20.962.5)

___________________________

Members of Visiting Committee,
Department of Economics
1950-1952

Chairman:

Clarence B. Randall
President, Inland Steel Company

Vice-Chairman:

David Rockefeller
Foreign Department, Chase National Bank

Members:

Henry W. Clark
Maritime Associates

Jack I. Straus (1951-52)
President, H. H. Macy Company

Sinclair Weeks
United Carr Fasteners Corp., Reed and Barton Corp.

Frederick C. Crawford
President, Thompson Products, Inc.

David F. Edwards
President, Saco-Lowell Shops

Devereux C. Josephs
Carnegie Corporation

Walter Lichtenstein
First National Bank of Chicago

Thomas S. Lamont (1950-51)
New York

David E. Lilienthal
formerly Head of Atomic Energy Commission

Edward R. Mitton (1951-52)
Jordan Marsh Company

Gilbert H. Montague
New York Lawyer

Edwin G. Nourse
formerly with Council of Economic Advisers

Ralph Robey
National Association of Manufacturers

Charles F. Rowley
Peabody, Brown, Rowley and Storey

Hermon Dunlop Smith
Marsh and McLennon, Insurance, Chicago

George Terborgh
Allied Machinery

Leo Wolman
Professor, Columbia University

Source: Harvard University Archives. Department of Economics, Correspondence and papers (UAV 349.11), Box 25, Folder “Visiting Committee, 1950-52”.

Images Sources:

(Left)  James B. Conant PageAtomic Heritage Foundation website.
(Right) Portrait of Trustee of the University of Chicago, Clarence B. Randall, from the University of Chicago Photographic Archive, apf1-03000-082, Hanna Holborn Gray Special Collections Research Center, University of Chicago Library.

Categories
Economics Programs Economists Harvard

Harvard. The Data Resources Inc. connection. Galbraith asks Eckstein, Feldstein, Jorgensen. 1972

 

“As Ed Mason tactfully hints, I’ve had enough lost causes for one year.”–Galbraith

In the following exchange of letters initiated by John Kenneth Galbraith in December 1972 we find multiple instances of seething rage barely concealed under veneers of formal academic politeness. Critical hiring and firing decisions regarding the subtraction of radical voices from the economics department faculty went overwhelmingly for the consolidation of mainstream economics earlier that month and Galbraith appears to have sought a vulnerability of this counterrevolution in its potential for conflicts of interest as he imagined coming from Otto Eckstein’s start-up, Data Resources, Inc. Eckstein’s response provides us with some interesting backstory to DRI. Feldstein and Jorgensen offered their witness testimony regarding this early episode in what would ultimately result in the so-called empirical turn in economics

But even after suffering this tactical defeat, Galbraith’s strategic point was to be confirmed by history:

“I do have one final thought. In accordance with the well-known tendencies of free enterprise at this level, one day one of these corporations is going to go down with a ghastly smash. It will then be found, in its days of desperation or before, to have engaged in some very greasy legal operations. The Department and the University will be held by the papers to have a contingent liability. It will be hard to preserve reticence then. It would have been better to have taken preventative action now.”

The conflict of interest cases brought by the U.S. Department of Justice in 2000 against economics professor Andrei Shleifer and the Harvard Institute for International Development resulted in a settlement that required Harvard to pay $26.5 million to the U.S. government.

_____________________________

On behalf of the Department,
Galbraith wants to know more about DRI

JOHN KENNETH GALBRAITH
HARVARD UNIVERSITY
CAMBRIDGE. MASSACHUSETTS

December 20, 1972

Professor Otto Eckstein
Littauer Center

Professor Martin S. Feldstein
1737 Cambridge Street

Professor Dale W. Jorgenson
1737 Cambridge Street

Dear Otto, Marty and Dale:

It will hardly be news that I have been deeply concerned over the several recent actions of the Department of Economics on appointments as well as the academically less consequential problem of the less than gracious response to those of us who have expressed alarm.

There is an impression, of which you will undoubtedly be sensitive, that the positions of some of those favoring the recent action could reflect, however subjectively and innocently, their corporate involvement in conflict with their academic responsibilities. I do not wish in any way to prejudge this matter or even to be a source of embarrassment. The problem does seem to me sufficiently somber so that in the interest of everyone you no less than the rest of us the circumstances should be clearly known. In this spirit I raise the following questions:

  1. Could you indicate the nature of Data Resources, Inc? I have reference to assets, sales, employees, services rendered, identity of corporate clients and charges.
  2. I believe it can fairly be assumed from general knowledge that the Corporation owes part of its prestige and esteem to association with members of the Harvard Department of Economics. The foregoing being so and reputation being a common property of the Department and Harvard University, could I ask as to your ownership or other interest or other participation of whatever sort and return?
  3. Has the Corporation employed students and nontenured members of the Department of Economics and would you indicate the names?
  4. Could I ask if you have participated in the past in the consideration of Harvard promotion of any such employees, consultants or people otherwise associated with the Corporation and in what cases?
  5. Could past service or inferior service or present or potential utility to the Corporation or extraneous judgment based on business as distinct from academic performance create, again perhaps subjectively, the possibility of a conflict of interest in your passing on Harvard promotions? How have you handled this conflict in the cases in which people with an association, past or present, with the Corporation have been up for Harvard promotion, always assuming that there have been such cases?
  6. In the recruiting of clients for the Corporation, what of the danger that they will be affected by the close relation between the Corporation and the Department? Specifically could there be effort, however subjective, to quell their fears? The radical economists come obviously to mind. But, as you are perhaps aware, even I am not a totally reassuring figure to many businessmen department with too many people of my viewpoint might also evoke alarm. Does safety here suggest that one with major corporate interest disqualify himself on all appointments?
  7. Is there a possibility — I by no means press the point that the kind of economics that serves corporate interest will take on an exaggerated importance when some of our ablest faculty members, and students are working on such problems?

Let me repeat that I ask these questions only for a clarification in which we share a common interest. I do not of course raise the more general question of outside activity. This would come with very poor grace from me — it is indeed the reason why I have sought not to be a charge on university resources,

Yours faithfully,

John Kenneth Galbraith

CC: Professor James S. Duesenberry

Dean John T. Dunlop

JKG:mih

_____________________________

Eckstein provides his answers to Galbraith’s “interesting questions”

Otto Eckstein
24 Barberry Road
Lexington, Mass. 02173
January 8, 1973

Professor J. Kenneth Galbraith
Department of Economics
Harvard University
207 Littauer Center
Cambridge, Mass. 02138

Dear Ken:

Pursuing the habits of a lifetime, you raise interesting questions in your letter of December 20th. Let me answer them by giving you an account of the origins and development of Data Resources, Inc., and of its relations to Harvard. I believe this will respond to all of your questions.

(1) Origins of DRI

As you know, my professional career has largely been devoted to the application of the techniques of economics to actual problems of the U.S. economy. After my most recent period of full -time government service in 1966, my views on the economy were sought by business and financial organizations. I quickly discovered that they made little use of macro economics or econometrics. The gap between macro and micro was unbridged. They typically ignored the overall situation. Econometrics, which always looked to me to be a very practical way to establish quantitative relationships, received little use and remained an academic plaything. I had already discovered in the government that even macro-decisions were made on the basis of very crude quantitative work, without the benefit of the thirty years of methodological development of econometrics.

In mid-1967, I had the idea that the technology of the time-sharing computer provided the missing link that would make it possible to use the modern techniques to improve private and public planning on a day-to-day basis. The time-sharing technology had the potential of overcoming the mechanical hurdles of programming, data punching, batch runs, etc. which had made econometrics a slow process open only to economists of exceptional mechanical aptitude. The time-sharing technology had the potential of bringing high quality data bases to researchers of providing them with the programs that would allow them to develop individual equations and to combine these equations into simulation models, and to evaluate their “satellite” models for historical analysis, contingency analysis and micro-forecasting. Such satellite models might encompass revenues and costs of their own industries or products, the detailed composition of unemployment, regional incomes, and the tax collections of governments.

These satellite models are constructed by users, at their own remote locations, combining their own data with the national data banks on the central computers. The programs allow the construction of the models and their on-line linkage to the centrally managed national models. Once the models are built, the particular company or government can quantitatively assess its own demand, costs, production, etc., assuming a particular macro-situation. It can see its own revenue and cost outlook assuming the central forecast, or alternatively what would happen if the economy should do better or worse. The micro-implications of changes in fiscal or monetary policy are also made apparent.

Besides making the tools that are our main stock-in-trade widely useable in the actual economy, the existence of such a system could accomplish these goals:

(1) There would be a rationally decentralized structure of information flows. The national data banks would be large and accessible, but local private information would remain where it belonged — in the confidential hands of the local analysts best equipped to use it.

(2) Analysis itself would be rationally decentralized. National forecasting could be done centrally with the use of lots of resources and with the benefit of an enormous data base and model collection. Micro forecasting would be done by the user organization itself.

(3) Micro-analysis would consider macro-environments as quantitative inputs. If the macro-forecasts are better than the crude assumptions previously made, the errors in micro-decisions should be reduced.

(4) As a result, the stability of the economy should be enhanced. There should be fewer and smaller mistakes in private and public economic decisions. Some of the benefits of indicative planning are realized without the political risks.

Once the basic ideas were clear, how was it to be done? The obvious possibilities were (1) a foundation financed project at Harvard; (2) persuade the government to undertake this work; (3) go to a large company  such as a computer manufacturer or bank; or (4) organize a new, small private enterprise. After some reflection, I decided that the new, small private enterprise form was the only suitable one. A Harvard project was ruled out immediately because of the poor experience with the Harvard Economic Barometers of the late 1920’s, an episode with which I was familiar from reading the archives of The Review of Economics and Statistics. Also, the system would require considerable operating staff for the computers, data banking, service and marketing. A university is not a good employer for such a staff nor a good working environment for these functions. I knew from my government experience that such a project was beyond the capacities of public agencies, at least in the United States, and budget stringency would have made federal funding unlikely, The large company would have posed difficult personal and political questions. Further, I felt that if the scheme were successful — and I had a good deal of faith in it — it could grow and reach its full potential by generating its own revenues. Finally, the idea of ultimately supporting my family from my main activities rather than “moonlighting” was attractive.

In 1968, Mitchell, Hutchins and Company, an investment firm with whom I was consulting, found the venture capital, an amount in seven figures. Donald Marron, its President, and I then co-founded DRI. The largest fraction of the capital was provided by First Security Corporation, an asset management group under the leadership of Mr. Robert Denison, a summa graduate of Harvard College and the Business School. The Board of Directors of the company are Mr. Marron, Mr. Denison, myself, and Mr. Stanton Armour, the Chairman of the Operating Committee of Mitchell, Hutchins.

The project required managers, econometricians, programmers, and computer experts. Mitchell, Hutchins managed the organization of the company, provided the initial business background and management, recruited personnel, etc. Dr. Charles Warden, previously special assistant to several chairmen of the CEA joined the company and took on many of its managerial burdens. Later on the company was organized into three divisions, each headed by a Vice-President.

Given the complexity and ambition of the scheme, I recognized that I needed the collaboration of the very best econometricians in terms of ideas, review and quality control. Mr. Marron and I, therefore, put together a founding consulting group, consisting of Jorgenson, Nerlove, Fromm, Feldstein, Hall and Thurow. This group made major contributions in the design stage. Today, the academic consultants mainly direct policy studies that DRI has been asked to undertake by government agencies and foundations. At all stages, the largest part of the work of developing and operating the DRI system and forecast was done by full-time professional employees of the company.

To help assure the widest application of the new techniques and to be able to offer alternative model forecasts, DRI entered into an agreement with the Wharton model group directed by Lawrence Klein. We continue to collaborate with them, and the Wharton model and its forecasts are maintained on the DRI computers. Subsequently, we have entered into arrangements with the model building group at the University of Toronto and with Nikkei, the sponsors of the Japan Economic Research Center.

As for the distribution of ownership, about half of the equity is in the hands of the institutions who provided the capital. Professional employees have ownership or options on another substantial fraction of shares, and my children and I own about a fifth of the shares. The academic consulting group has about 5% of the shares, received at the time of the founding of the company. All of the stock is restricted; it is not registered with the SEC and hence not saleable. The academic consultants are paid on a per diem basis as they actually spend time. In order to give the company a better start, I did not take any pay in the first three years; last year I began to receive a modest compensation.

(2) The Status of DRI Today

On the whole, my hopes and aspirations for DRI have been realized The economic data bases are the most comprehensive in existence and their accuracy is unquestioned. The econometric models have advanced that art in certain respects. The forecasts have been good and are now followed and reported quite widely. The people — management, research economists, service consultants, data processing and programming experts, and marketing — are capable and the organization is strong. While it inevitably takes time for new concepts and techniques to gain acceptance and be widely adopted, more than half of the fifty largest industrial companies and a large fraction of the financial institutions utilize the DRI system. Every major government agency involved in macro economic policy as well as every major data producing government agency is a user of the DRI system. The research environment created by the DRI data banks, software, models and computers has proved so attractive that even organizations with considerable internal facilities find it useful to have access. DRI as an organization has no political views, though individuals associated with the company can take any position they wish.

Our system has also been used by ten universities and colleges and we have just begun to develop special services for the state governments. As DRI is becoming better known and our communications network to our computers spreads to cover a far greater number of communities, we expect that more colleges and universities will find it possible to take advantage of these research facilities.

The company reached the break-even point in the twentieth month of operation after expending the larger part of the venture capital to create the initial version of the DRI system. It is now moderately profitable and earnings are advancing rapidly. Thus far, the capitalists have earned no return of dividends or interest. They have been extraordinarily forbearing in not pressing for quick returns, preferring to let the company use all of the resources in these early years to bring the DRI concept to full fruition. The probabilities are good that the investors will be handsomely rewarded over the next few years. Having taken the risk and waited, they will have earned their return.

(3) The Relation of DRI to Harvard University

Recognizing the sensitivity of this issue from the beginning, I have made sure that Data Resources produced a flow of benefits to Harvard and that Harvard would not provide resources to DRI. The Board of Directors, heavy with Harvard alumni, formally instructed me early in our development to provide free use of the DRI system to Harvard students. Quite a few have done so, including students on my small NSF project on prices and wages. This Fall, for the first time, I have a graduate working seminar in econometric model building. Each of the seven students enrolled is building his own model, simulating it, and writing a paper. The projects include the first econometric model of Ghana, a small scale two-country model of Canada and the United States, an exercise in policy optimization using the DRI model, a study to use macro models to estimate the changing distribution of income, a study of tax incidence using translog production functions, and a model of Venezuela. If this experimental seminar is successful, a lot more can be done, of course.

In terms of relations with professors, Feldstein and Jorgenson were members of the original academic consulting group, along with professors at MIT, Chicago, Brookings and Wharton. I direct and take responsibility for the DRI forecasts, working with full -time employees. The others have focussed on policy studies, including three major studies for the Joint Economic Committee which received considerable attention. They have also done studies for the U.S. Treasury, the Ford Foundation, etc. These studies have not been a significant source of profit to the company, but they surely help to build Data Resources as an authoritative source of economic analysis and serve the public interest.

DRI has had very limited relations with the non-tenured faculty in the Harvard Economics Department. We cooperated with the Department in January 1969 to make it possible for Barry Bosworth to assume his appointment a semester early when he wished to leave the Council of Economic Advisers. He did some useful research that spring and summer, most of which reached fruition in his subsequent papers at The Brookings Institution. His half-time support was transferred to a project at Harvard after one semester. Mel Fuss collaborated in the early stages of our analysis of automobile demand sponsored by General Motors. Bill Raduchel has done some consulting in the programming area with us, but this was always was a very minor part of his activities. While it would be improper to recount the precise role of myself or Feldstein and Jorgenson in the promotion considerations of these three men, it is perfectly obvious and easily documented that there is no substantive historical issue of DRI considerations entering into Harvard appointments. Bosworth went to Brookings before his appointment came up; Fuss and Raduchel were not promoted.

Perhaps this is the point to digress on my philosophy on Harvard promotions. I believe that assistant professors should be selected on the basis of professional promise, their potential contribution to the undergraduate teaching program and whatever publication record they already possess. Promotion to associate professor should mainly be based on research accomplishments as well as teaching performance, with both prerequisites. I have always strongly felt that collaboration in the research projects of senior professors should be given no weight in non-tenured appointments because of the considerable risk that the Harvard appointment thereby becomes a recruiting device for the personnel of these projects. In my years at Harvard, I have never asked the Department to appoint anyone whose presence would be useful to me, and I never will make such a request. To the best of my knowledge, Feldstein and Jorgenson have pursued the same policy. I recommend adoption of procedures that would assure that all of us avoid such appointments.

There are more intangible relations between DRI and Harvard which are hard to assess and easy to exaggerate. If I did not possess a professional reputation which has been enhanced by my professorship here my career would have been different, and I might not have received my extraordinary opportunities of public service. As far as the development of DRI is concerned, my greatest institutional indebtedness is to the Council of Economic Advisers. It was this experience which made me appreciate the importance of accurate and quick information and of the tremendous potential of using econometrics to bridge the gap between macro- and micro-economics. As far as the relations with our private and public clients are concerned, a sophisticated group containing numerous Harvard graduates, they understand perfectly well the tremendous diversity of people and ideas present at Harvard. They know that Harvard has no institutional position on political questions or on the merits or demerits of the existing social, political or economic system. It is also clear to them that Data Resources is a totally distinct entity. I am not responsible for your views and you will not be tainted by mine.

Your final question, whether “the kind of economics that serves corporate interest will take on an exaggerated importance when some of our ablest faculty members and students are working on such problems” is a deep philosophical one which I can only attempt to answer in this way. The Harvard Economics Department has always contained individuals with widely varying concepts of their role in life and preferences in their professional activities. Compared to its historical position, the Department at this time is exceptionally heavy in abstract theory and methodology, and in social philosophy and criticism of the existing order. I represent a different point of view that has always been common in our department. It is my aim to apply economics to the country’s problems in the belief that the existing system can be made to meet the needs of the good society. The development of Data Resources is my current personal expression of this philosophy.

Sincerely yours
[signed] Otto
Otto Eckstein

OE/gc

_____________________________

Feldstein reports being a satisfied user of DRI services

HARVARD UNIVERSITY

MARTIN S. FELDSTEIN
Professor of Economics

1737 CAMBRIDGE STREET, 617
CAMBRIDGE, MASSACHUSETTS 02128

January 9, 1973

Professor J. K. Galbraith
Department of Economics
Harvard University
Littauer 207

Dear Ken:

Although I was surprised by your letter, I am happy to describe my relations with Data Resources. I have been an “economic consultant” to DRI since it was organized. I would describe both the amount of work that I have done and my financial interest as very limited. Last year, my only DRI work was a study of the problem of unemployment that I did for the Congressional Joint Economic Committee. The Committee contracted with DRI for the study. DRI provided the use of the DRI model and data bank and the special computing facilities. Professor Robert Hall of MIT, another DRI consultant, worked on the study for a few days. The study, Lowering the Permanent Rate of Unemployment, was used as the background for hearings in October and will be published by the Committee this year. I am enclosing a copy for your interest. I might also note that although the work on this for DRI is now complete, I am planning to continue on my own to do research on some of the problems that I examined in this study. A graduate student who helped me during the summer became so interested in some of the questions of labor force participation that he is considering doing his thesis on that subject.

Before last year I worked on developing the financial sector of the Data Resources model. The basic work here was building a bridge between the usual Keynesian analysis and the Fisherian theory with its emphasis on the expected rate of inflation. My work here started as direct collaboration with Otto Eckstein; we published a joint paper, “The Fundamental Determinants of the Interest Rate,” in the 1970 Review of Economics and Statistics. This research led me to consider the importance of expected inflation in all studies of the impact of interest rates; I described my work on this in “Inflation, Specification Bias, and the Impact of Interest Rates” (Journal of Political Economy, 1970). Although further work on the financial sector is now done primarily by members of the DRI full-time staff, I did some work in 1971 on extending the analysis of expectations and testing alternative econometric models of expectations. This work is described in a recent paper, “Multimarket Expectations and the Rate of Interest” with Gary Chamberlain, that has been submitted for publication.

I have described my DRI studies in such detail to give you a sense of both the substance and nature of the work. It has been scientific research on substantively and technically interesting questions of macroeconomics and macroeconomic policy. I have also found the access to the DRI facilities, particularly the macroeconomic model system and data bank, to be useful in my other research and teaching.

I cannot believe that my association with DRI could create any of the problems that you indicate in your questions 5, 6 and 7. I believe that Otto is writing to you about the specific points that you raised about DRI in your questions 1 through 4. I hope that all of this material reassures you about the relations between DRI and members of our department.

Please call me if you have any further questions,

Sincerely,
[signed] Marty
Martin S. Feldstein

MSF:JT

Enclosure

_____________________________

Galbraith to Feldstein: You did not address my concern about “problems of conflict of interest”

January 19, 1973

Professor Martin S. Feldstein
Room 617
1737 Cambridge Street

Dear Marty:

Many thanks for your detailed — and good-humored — response. I’m grateful also for the JEC Study of which Otto spoke and which I am taking to Europe for my own reading. I have taken the liberty of giving a copy of your letter to Ed Mason who, as you perhaps know, is making a study of this whole problem.

As you can guess, I am untroubled by work done directly or through DRI for the government. I am concerned about the problems of conflict of interest that seem to me to arise when a corporation which owes its esteem to members of our Department markets profit-making services to other corporations. But this is something on which I should like to reserve comment until Ed Mason has come up with his conclusions.

Yours faithfully,

John Kenneth Galbraith

JKG:mjh

_____________________________

Jorgenson: I think you are barking up the wrong tree

HARVARD UNIVERSITY
DEPARTMENT OF ECONOMICS

January 22, 1973

DALE W. JORGENSON
Professor of Economics

1737 CAMBRIDGE STREET, ROOM 510
CAMBRIDGE, MASSACHUSETTS 02138
(617) 495-4661

Temporary Address until 6/30/73:
Department of Economics
Stanford University
Stanford, California 94305

Professor John Kenneth Galbraith
Littauer 207
Harvard University
Cambridge, Massachusetts 02138

Dear Ken:

Many thanks for your letter of December 20 and your note of December 21. Let me take this occasion to thank you for the copy of your AEA Presidential Address you sent to members of the Department. It was a masterpiece of the genre and will be long remembered by its readers. I am very sorry that I was unable to attend your oral presentation at Toronto.

I share your deep concern over recent actions of the Department of Economics on non-tenure personnel, even though our views on these matters do not always coincide. In view of the strong feelings involved I found the discussion to be remarkably free of personal considerations. I hope that I have not been a party to what you describe as a less than gracious response to vour own views. If I have, I hope that you will accept my apologies.

Since your letter is addressed to Otto Eckstein, Martin Feldstein and myself, I will limit this response to my own role in DRI. I am a stockholder and consultant to DRI and have been for almost four years. In my work for DRI, I have acted as a consultant to several U.S. government agencies and to the Ford Foundation. I have had only one corporate client for my services. My main current activity for DRI is a study of energy policy for the Ford Foundation.

DRI provides a unique environment for certain types of research in applied econometrics. My current work on energy policy would be infeasible without the DRI system. The computer software, computerized data bank, and econometric forecasting system have been indispensable in modeling the energy sector and in studying the effects of economic policies related to energy. The facilities available at DRI have reduced the burden of data processing and computation for econometric model-building by several orders of magnitude.

To my mind the two most important features of the DRI system are its high quality from the scientific point of view and its ability to assimilate the results of research and to make them available for routine application. The data bank is unparalleled in scope and reliability and is constantly expanding as new sources of data are made available. The computer software package is highly sophisticated and is under continuous development as new econometric methods are designed. The forecasting system is the core of DRI’s operations and has undergone a process of improvement and extension that has continued up to the present.

The performance of the DRI system is the main source of attraction for DRI’s clients. This is certainly the case for my study of energy policy. You raise a general question about the concerns of DRI’s clients and the views of members of Harvard’s Department of Economics. In my experience there is no connection, either positive or negative. The clients of DRI are buying the services of DRI. As I have already indicated, this is a rather unusual product, unavailable at any university economics department, including Harvard’s.

On the issue of non-tenured members of the Department of Economics who are also employee-consultants of DRI, I have not employed any non-tenured members of the Department in my work for DRI, as I indicated in our telephone conversation. I find it difficult to envision circumstances in which any conflict of interest related to junior appointments could arise from my DRI association. There have been no such circumstances in the past.

I hope that these observations help to clarify the issues you raise

Yours sincerely,
[signed] Dale
Dale W. Jorgenson

DWJ: cg

cc: E. Mason, J. Dunlop, H. Rosovsky, R. Caves, J. Duesenberry, O. Eckstein, M. Feldstein

_____________________________

Galbraith back to Jorgenson: we need to avoid even the appearance of a  “conflict of interest”

Gstaad. Switzerland
February 13, 1973

Professor Dale W. Jorgenson
Department of Economies
Stanford University
Stanford, California 94305

Dear Dale:

Many thanks for your letter and for your nice comments. I hope life goes well for you at Stanford. I am writing this from Switzerland where I am on the final pages of what I intend shall be my last major effort on economics. When I get tired I propel myself across the snow and think how good the mountains in the winter would be in a world where one did not feel obliged to take exercise.

I must say that my attention after writing was shifted to yet another of our corporations of which, to my annoyance, I was unaware. It functions currently, I gather, as a subsidiary of the antitrust problems of IBM.

I do feel that there are serious problems here. Participation in the management of the Department, especially in the selection and recruitment of personnel, and in the management of a profit-making enterprise are bound to involve if not the reality of conflict of interest then the appearance of conflict. Appointments, it will be held, are influenced by what influences corporate customers or needs. This must be avoided. It is especially clear if the corporation sells such services as antitrust defense. But it is also the case if the corporation becomes large and successful —, as I would judge, DRI is certain and deservedly to be.

The proper course, as I have suggested to Ed Mason and informally to Otto, is not to deny any professor the right to participation in a profit-making enterprise. Rather it is to separate the two management roles. A man should be free to have an active ownership role in a corporation or an active position in Department management. He should not do both. This would obviate problems of conflict or seeming conflict and protect the positions of all concerned. Needless to say, I would have the same rule apply to all.

Yours faithfully,

John Kenneth Galbraith

JKG:mjh

cc: E. Mason, J. Duesenberry, O. Eckstein, M. Feldstein, R. Caves, H. Rosovsky, F. Ford

_____________________________

“Economics Dept. Reports On Faculty’s Outside Ties”
by Fran R. Schumer. Harvard Crimson, March 20, 1973

A committee in the Economics Department reported yesterday that business connections between Economics professors and outside corporations do not interfere with hiring decisions and teaching practices.

James S. Duesenberry, chairman of the three-man committee, said yesterday that business ties do not impose a conservative bias on the Department’s hiring practices and do not limit the faculty’s teaching time.

Complaints

The committee’s investigation was prompted by complaints raised last term by John Kenneth Galbraith, Warburg Professor of Economics.

Galbraith attributed the Department’s “conservative hiring practices” to faculty members’ ties with business firms. “The fact that the Department sells its services to American business firms biases its administrative decisions,” Galbraith said.

Despite the committee’s negative findings, Otto Eckstein, professor of Economics and president of Data Resources Inc., a consulting firm, has requested to go on half-time status at Harvard, effective September 1.

Eckstein said yesterday that his decision resulted from Galbraith’s complaints and a new rule prohibiting professors from spending more than one day a week consulting. The rule, previously implicit, was formally written into University law this year.

Galbraith voiced objections to faculty members’ business ties several weeks after the Department’s decision last December not to rehire two radical economists.

At that time, Galbraith told Duesenberry that “business ties necessarily impair the faculty’s ability to impartially judge economists, especially radical economists.”

Galbraith also complained that the Department’s decision last December not to promote William J. Raduchel, assistant professor of Economics, was based on the quality of Raduchel’s work for an outside Resources had little influence on the consulting firm and not on his research and teaching abilities in the Department.

Raduchel is a consultant for Data Resources Inc. and is also a sectionman for Galbraith’s course, Social Science 134, “The Modern Society.”

The committee, composed of Duesenberry, Arthur Smithies, Ropes Professor of Political Economy, and Richard E. Caves, Stone Professor of International Trade, reported last January that Raduchel’s work for Data Resources had no influence on the Department’s decision.

The committee also reported that outside ties do not prejudice the Department’s hiring decisions and do not interfere with normal administrative functioning.

The committee reported its findings only to Duesenberry, the chairman of the Economics Department. Committee members refused to comment on how they investigated the problem.

Duesenberry attributed Galbraith’s objections to the Department’s decision not to promote Raduchel. “Galbraith is annoyed because his boy didn’t get promoted,” he said.

Raduchel told The Crimson last month that he was satisfied with the Department’s decision not to promote him. He said that the decision had “nothing to do with my connection to Data Resources, and was based on my academic work.”

Eckstein agreed with Duesenberry’s conclusion that Raduchel’s work at Data Resources had little influence on the Department’s decision.

Explaining his own position at Data Resources Inc. Eckstein said that his case is no different than that of other faculty members who do consulting work.

Currently, at least three senior faculty members and one junior faculty members do consulting work at Data Resources.

Eckstein described consulting work an inevitable product of Harvard’s hiring policies. “Harvard naturally attracts people who get involved in the outside world,” he explained.

He said that he has a “clear conscience” about the work he is doing at Harvard.

_____________________________

Galbraith to Chairman Duesenberry:

Gstaad, Switzerland
March 27, 1973

Professor James S. Duesenberry
Littauer M-8

Dear Jim:

Herewith some good-humored thoughts on our final talk the other day about our corporate affiliates. As you request, I will now leave the problem to the President, Steiner and whomever.

  1. Although both you and Henry Rosovsky had earlier expressed discomfort about our corporation and some action now seems in prospect, you say I’m severely viewed for raising the issue. Isn’t this a little hard? The important thing, I suggest, is to get things right. However, although given my sensitive soul it has been difficult, I have steeled myself over the years to the idea of not being universally loved.
  2. You say that the bias from combining business entrepreneurship with professorial activities in the eye of some of our colleagues is not greater than that deriving from my (or Marc Roberts’) support of George McGovern. I somehow doubt that the faculty would agree. There is indication of difference, I think, in the way one reacts. I do not find myself shrinking especially from identification even with anything now so widely condemned as the McGovern campaign. I detect a certain desire to avoid public discussion of our corporations.
  3. In keeping with the desire for reticence, I told Ed Mason I wouldn’t talk with the press. The Crimson tells me that you have explained that I raised the issue only out of pique over the non-promotion of Raduchel. Isn’t this a bit one-sided? However, beyond denying any such deeply unworthy motive, I’ll stick to my agreement, always reserving the right of self-defense.
  4. As to my motives, so far as I can judge them, I did feel that Raduchel got judged on his corporate work, while — as Smithies and I both complained — there was no consultation with those who best knew about his teaching. His teaching has been very good. I suggest that we are always in favor of improving undergraduate teaching in principle but not in practice. Also I do not agree that he was unpromotable. He has a lively, resourceful mind and has worked hard for the University and the students. I think him far, far better than the dull technicians we do carry to the top of our nontenured ranks, possibly even beyond.
  5. But, as I probe my soul for the purest available motive, it was not Raduchel. I simply think that, when a professor speaks or acts on a promotion, we should know that he is doing it as a professor and not as a businessman.
  6. I had thought that the separation of our business arrangements from the Department management might be a solution, with the proposed withdrawal of voting rights from the aged as a precedent. This, I gather, will not wash, so I subside. As Ed Mason tactfully hints, I’ve had enough lost causes for one year.

I do have one final thought. In accordance with the well-known tendencies of free enterprise at this level, one day one of these corporations is going to go down with a ghastly smash. It will then be found, in its days of desperation or before, to have engaged in some very greasy legal operations. The Department and the University will be held by the papers to have a contingent liability. It will be hard to preserve reticence then. It would have been better to have taken preventative action now.

Conforming to your wish that I restrict communications on this subject, I’m not circulating this letter. But would it trouble you If I added it discreetly to the file in the President’s office? Do let me know.

Yours faithfully,

John Kenneth Galbraith

JKG:mjh

Source: John F. Kennedy Presidential Library. John Kenneth Galbraith Personal Papers. Series 5 Harvard University File, 1949-1990. Box 526. Folder “Harvard Dept. of Economics. Discussion of appointments, outside interests and reorganization, 1972-1973 (1 of 2)”.

Image Sources: John Kenneth Galbraith (1978), Harvard University Archives; Otto Eckstein (April 1969), Harvard University Archives; Martin Feldstein (ca. 1974), Newton Free Library, Digital Commonwealth, Massachusetts Collections Online; Dale Jorgenson. (1968). John Simon Guggenheim Memorial Foundation.

Categories
Exam Questions Harvard

Harvard. General Examination in Microeconomic Theory. Spring, 1993

Economics in the Rear-view Mirror has been provided a copy of nearly all the 1990s general exams in micro- and macroeconomic theory from Harvard through the collegial generosity of Minneapolis Fed economist Abigail Wozniak. With this post you now have the Spring 1993 graduate general exams in microeconomic theory.

While these exams lie outside of my personal comfort zone as a historian of economics (1870-1970), for fledgling historians of economics of today and tomorrow these are indeed legitimate historical artifacts definitely worth transcription. I am rather slow in digitizing them because transcription of mathematics for this blog requires latex inserts. Latex expressions involve considerably longer roundabout production than the application of my talents for touch-typing/OCR to non-mathematical text. Patience! The Rest is Yet to Come! 

________________________________

Previously Transcribed Harvard Graduate General Exams

Spring 1989: Economic Theory

Spring 1991: MicroeconomicsMacroeconomics

Spring 1992: Micro- and Macroeconomics

Fall 1992:  Micro- and Macroeconomics

________________________________

Graduate Microeconomic Theory Sequence, 1992-93

Economics 2010a. Economic Theory

Michael D. Whinston and Eric S. Maskin

Covers the theory of individual behavior including the following topics: constrained maximization, duality, theory of the consumer, theory of the producer, behavior under uncertainty, consumer choice of financial assets, externalities, monopolistic distortions, game theory, oligopolistic behavior, asymmetric information.

Prerequisite: Economics 2030 or equivalent; can be taken concurrently.
Half course (fall term). Tu., Th., 10-11:30.

Economics 2010b. Economic Theory

Andreu Mas-Colell and Stephen A. Marglin

General equilibrium, stability, pure and applied welfare economics, uncertainty, descriptive and optimal growth theory, income distribution, methodology.

Prerequisite: Economics 2010a.
Half course (spring term). Tu., Th., 10-11:30.

Source: Harvard University, Faculty of Arts and Sciences. Courses of Instruction 1992-1993, p. 248.

________________________________

HARVARD UNIVERSITY
DEPARTMENT OF ECONOMICS

Economics 2010b: FINAL EXAMINATION and
GENERAL EXAMINATION IN MICROECONOMIC THEORY

Spring Term 1993

For those taking the GENERAL EXAM in microeconomic theory:

  1. You have FOUR hours.
  2. Answer a total FIVE questions subject to the following constraints:

— at least ONE from Part I;
— at least TWO from Part II;
EXACTLY ONE from Part III.

For those taking the FINAL EXAMINATION in Economics 2010b (not the General Examination):

  1. You have THREE HOURS
  2. Answer a total of four questions subject to the following constraints:

— DO NOT ANSWER ANY questions from Part I;
— at least TWO from Part II;
— at least ONE from Part III.

PLEASE USE A SEPARATE BLUE BOOK FOR EACH QUESTION

PLEASE PUT YOUR EXAM NUMBER ON EACH BOOK

Part I (Questions 1 and 2)

  1. Suppose that there are J firms producing good \ell differentiable cost function c(w,q) where w is a vector of input prices and q is the firm’s output level. The differentiable aggregate demand function for good \ell is x(p), where p is good \ell’s price. Assume c(w,q) is strictly convex in q and that (p)≤0. Also assume that partial equilibrium analysis is justified.
    1. Suppose that all factor inputs can be adjusted in the long-run, but that input k cannot be adjusted in the short-run. Suppose that we are initially at an equilibrium where all inputs are optimally adjusted to the equilibrium level of output \bar{q} and factor prices \bar{w} so that, letting z_{k}\left( \bar{w} ,\bar{q} \right) be the conditional factor demand function for factor k, we have z_{k}=z_{k}\left( \bar{w} ,\bar{q} \right). What can be said about the short-run versus long-run output response of the firm to a differential change in the price of good \ell? What does this imply about the short-run versus long-run equilibrium response of p to a differential exogeneous shift in the demand function (hold the number of firms fixed in both cases)? (Hint: Define a short-run cost function c_{s}\left( w,q,z_{k}\right)  giving the minimized cost of producing output q given factor prices w when factor k is fixed at level z_{k}).
    2. Now suppose that all factor inputs can be freely adjusted. Give the weakest possible sufficient condition, stated in terms of marginal and average costs and/or their derivatives, that guarantees that if the price of input k\left( w_{k}\right) marginally increases, then firms’ equilibrium profits decline for any demand function x\left( \cdot \right) with x^{\prime }\left( \cdot \right)  \leq 0. Show that if your condition is not satisfied, then there exist demand functions such that profits increase when the price of input k increases. What does your condition imply about the firm’s conditional factor demand for input k?
  2. A. Consider a one-shot two-player game in which player 1 has a set of possible moves M1 (with n1 elements) and player 2 has a set of possible moves M2 (with n2 elements). Players move simultaneously. How many strategies does each player have?

B. Now suppose that the game is changed so that player 1 moves before 2, and 2 observes 1’s move, but that the game is otherwise the same as before. That is, the sets of moves are still M1 and M2, and player 1’s and 2’s payoffs as functions of moves \psi_{1} \left( m_{1},m_{2}\right) \text{ and } \psi_{2} \left( m_{1},m_{2}\right), respectively, are unchanged. How many strategies does each player have in the altered game?

C. The game of part B may have multiple subgame-perfect equilibria. Show, however, that, if this is the case, there exist two pairs of moves \left( m_{1},m_{2}\right)\text{ and } \left( m^{\prime }_{1},m^{\prime }_{2}\right) (where either m_{1}\neq m^{\prime }_{1}\text{ or } m_{2}\neq m^{\prime }_{2} ) such that either

(*) \psi_{1} \left( m_{1},m_{2}\right)  =\psi_{1} \left( m^{\prime }_{1},m^{\prime }_{2}\right)
or
(**) \psi_{2} \left( m_{1},m_{2}\right)  =\psi_{2} \left( m^{\prime }_{1},m^{\prime }_{2}\right).

D. Suppose that, for any two pairs of moves \left( m_{1},m_{2}\right)\text{ and } \left( m^{\prime }_{1},m^{\prime }_{2}\right)  such that m_{1}\neq m^{\prime }_{1}\text{ or } m_{2}\neq m^{\prime }_{2}, (**) is violated, i.e., \psi_{2} \left( m_{1},m_{2}\right)  \neq \psi_{2} \left( m^{\prime }_{1},m^{\prime }_{2}\right). In other words, player 2 is never indifferent between pairs of moves. Suppose that there exists a pure-strategy equilibrium in the game of part A in which \pi_{1} is player 1’s payoff. Show that in any subgame-perfect equilibrium of part B, player 1’s payoff is at least \pi_{1}. Would this conclusion necessarily hold for any Nash equilibrium of part B? Why or why not?

E. Show, by example, that the conclusion of part D may fail if either

(a) \psi_{2} \left( m_{1},m_{2}\right)  =\psi_{2} \left( m^{\prime }_{1},m^{\prime }_{2}\right)  holds for some pair \left( m_{1},m_{2}\right)  ,\left( m^{\prime }_{1},m^{\prime }_{2}\right) with m_{1}=m^{\prime }_{1}\text{ and } m_{2}=m^{\prime }_{2}; or

(b) we replace the phrase “pure-strategy equilibrium” with “mixed-strategy equilibrium.”

Part II (Questions 3, 4, & 5)

QUESTION 3 (General Equilibrium with Gorman Preferences)
(20 points)

Suppose you have a population of consumers i = 1,….,I. Ever consumer i has an endowment vector of commodities \omega_{i} \in R^{I} and preferences expressed by an indirect utility function v_{i}\left( p,w_{i}\right).

(a) (5 points)

Let \left(\bar{x}_{1},\cdots,\bar{x}_{I}\right) be a Pareto optimal allocation. The utility levels of this allocation are \left(\bar{u}_{1},\cdots,\bar{u}_{I}\right). The second welfare theorem asserts the existence of a price vector \bar{p} and wealth levels \left(\bar{w}_{1},\cdots,\bar{w}_{I}\right) supporting the allocation. What does this mean? Express \left(\bar{u}_{1},\cdots,\bar{u}_{I}\right) in terms of the indirect utility functions.

Assume for the next two parts of this question (b and c) that the indirect utility functions take the (Gorman) form v_{i}\left( p,w_{i}\right)  =a_{i}\left( p\right)  +b\left( p\right)  w_{i}. Note that b\left(\cdot\right) does not depend on i. In the following, neglect always boundary allocations. Use of pictures is permissible and helpful.

(b) (5 points)

Show that for the above family of utility functions all the Pareto optimal allocations are supported by the same price vector.

(c) (5 points)

Use the conclusion of part (b) to argue that the Walrasian equilibrium allocation is unique. (Assume preferences are strictly convex.)

For the last part of the question (d) assume that indirect utilities are of the form v_{i}\left( p,w_{i}\right)=b_{i}\left(p_{i}\right)w, that is, the preferences on commodity bundles are homothetic (but possibly different across consumers).

(d) (5 points)

Argue by means of an Edgeworth box example (or in any other way you wish!) that the multiplicity of Walrasian equilibria is possible even if preferences are restricted to be homothetic.

QUESTION 4 (Revelation of Information Through Prices)
(20 Points)

Suppose there are two equally likely states s_{1},s_{2} and two traders. In each state there is a spot market where a good is exchanged against numeraire. The utilities of the two traders are (the second good is the numeraire):

STATE 1 STATE 2
TRADER 1 2 ln x11x21

4 ln x11 + x21

TRADER 2

4 ln x12 – x22

2 ln x12 + x22

The total endowment of the first good equals 6 in the first state and 6+\varepsilon    in the second state. All the endowments of this good are received by the second trader. Assume that the endowments of numeraire for the two traders are sufficient for us to neglect the possibility of boundary equilibria. The price of the numeraire is fixed to 1 in the two states. The prices of the non-numeraire good in the two states are denoted \left( p_{1},p_{2}\right)  .

(a) (5 points)

Suppose that when uncertainty is resolved both traders know which state of the world has occurred. Determine the spot equilibrium prices \left(\hat{p}_{1}\left(\varepsilon\right) ,\hat{p}_{2}\left(\varepsilon\right)  \right) in the two states (as function of the parameter \varepsilon).

(b) (5 points)

We assume now when a state occurs Trader 2 knows it while Trader 1 remains uninformed (i.e. s/he must keep thinking of the two states or equally likely). Under this information set up determine the spot equilibrium prices \left( \bar{p}_{1}\left(\varepsilon\right) ,\bar{p}_{2}\left(\varepsilon \right)\right) in the two states.

(c) (5 points)

We are as in (b), except that now we allow Trader 1 to deduce the state of the world from prices. That is, if p_{1}\neq p_{2} then Trader 1 is actually informed while if p_{1}=p_{2}, s/he is not informed. A system of equilibrium spot prices \left( p^{\ast }_{1}\left(\varepsilon\right) ,p^{\ast }_{2}\left(\varepsilon\right) \right) is a rational expectation equilibrium if at the equilibrium Trader 1 derives information from \left( p^{\ast }_{1}\left(\varepsilon\right) ,p^{\ast }_{2}\left(\varepsilon\right) \right) in the manner described. Let \varepsilon \neq 0. Exhibit a rational expectations equilibrium. Comment.

(d) (5 points)

Show that if \varepsilon = 0 then there is no rational expectations equilibrium.

QUESTION 5 (20 Points)

There are three participants in a public good decision problem with two outcomes. If the public good project is not carried out then the utility is zero for everybody. If it is carried out then the utility is 3 for the “project-lovers” and -1 for the “project-haters.” The cost of the project is zero.

We consider first the following decision mechanism. People are asked if they are PL (project-lovers) or PH (project haters). If at least one participant announces PL the project is carried out and the self-declared PH are exactly compensated for their loss. The resources for the compensation comes from a tax imposed on the self-declared PL (equal across them).

(a) (5 points)

Show that the above mechanism is not straightforward. Define your terms.

(b) (5 points)

Suppose now that participants know each others characteristics (i.e. if they are project-lovers or project-haters). Consider the situation where everybody self-declares truthfully. Argue that this is an equilibrium (i.e. it does not pay to any participant to deviate) if there is one but not if there are two PLs. Which are the equilibrium situations in the latter case?

We now change the set-up somewhat. Suppose that the designer knows how many PLs there are and that the participants know that the designer knows (or, simply you can assume that both designer and participants have this information). Say that the number of PLs is \alpha \in \left( 1,2,3\right)  . (Hence there is at least one PL.) Then the decision mechanism is as above except that for the project to be carried out it is now required that at least \alpha self-declare as PL.

(c) (5 points)

Show that for this mechanism it does not always pay to self-declare truthfully (that is, the truth is not a dominant strategy).

(d) (5 points)

Suppose that it is understood (Precisely, it is common knowledge) that no participant will ever use a dominated announcement. Show then that it cannot hurt to self-declare truthfully (technically, the truth is dominant after one round of deletion of dominated strategies. There is a subtle point here—that you may want to discuss—namely, if “dominated” should be understood as “weakly dominated” or “strongly dominated.” The distinction does not matter for the case \alpha =1 but it does for the case \alpha =2.)

Part III (Questions 6 and 7)

  1. (a) How does the following idea (or vision, in Schumpeter’s sense of the term) get reflected in the neo-Keynesian model presented in this course?

…there is a subtle reason drawn from economic analysis why…faith may work. For if we act consistently on the optimistic premise, this hypothesis will tend to be realized; whilst by acting on the pessimistic premise, we keep ourselves for ever in the pit of want. (Keynes, Essays in Persuasion, pp. vii-viii)

(b) Why does Knight’s dictum [following] fail to characterize the neo-Keynesian model?

…competition among even a very few [entrepreneurs]will raise the rate of contractual returns [wages] and lower the residual share [profits], if they know their own powers. If they do not, the size of their profits will again depend on their “optimism,” varying inversely with the latter. (Knight, Risk, Uncertainty, and Profit, p. 285.)

(c) Is it true, as Joan Robinson once wrote, that in a neo-Keynesian conception of the world businessmen are free to make the rate of profit anything they wish?

(d) More generally, how can investment demand be exogenous in a model where income and expenditure must be equal as a condition of equilibrium? What features of the theory allow investors’ preferences and investment demand to play a role in neo-Keynesian theory which differs from the role played by consumers preferences and consumption demand in neoclassical theory?

  1. Economic theories are, among other things, theories of knowledge—implicitly if not explicitly. What is the neoclassical theory of knowledge? Which do you regard as the more serious of the many objections to this theory of knowledge? Why in your view has the theory been able to survive the objections?

Source: Department of Economics, Harvard University. Past General Exams, Spring 1991-Spring 1999, pp. 84-88. Private copy of Abigail Waggoner Wozniak.

 

Categories
Columbia Curriculum Economics Programs Harvard Yale

Columbia. Plans for Graduate Classes and Creation of a School for Civil Service Preparation. Burgess, 1880

 

In the beginning was the pitch. On June 7, 1880 the Board of Trustees of Columbia College passed resolutions to establish the School of Political Science (within which the field of political economy was embedded). The School of Political Science would open for students beginning with the 1880-1881 academic year. The pitch and plan for a “School of Preparation for the Civil Service” of February 1880 by the School of Political Science founder John W. Burgess  is transcribed below.

From this document is clear that in Burgess’  initial vision the principal mission of the future Faculty of Political Science would be to “prepare young men for the duties of public life”. So one can think of the ancestor faculty from which today’s department of economics at Columbia University descended was actually a School of Public Policy in which History, Law, and Philosophy constituted the supporting disciplinary pillars. 

__________________________

OUTLINE OF A PLAN
FOR THE
INSTRUCTION of GRADUATE CLASSES
FOR THE
EXTENSION OF THE ELECTIVE SYSTEM OF STUDY
IN THE
UNDERGRADUATE DEPARTMENT.
AND FOR THE CREATION OF
A SCHOOL OF PREPARATION
FOR THE
CIVIL SERVICE

Macgowan & Slipper, Printers, 30 Beekman Street, New York.

__________________________

COLUMBIA COLLEGE

GRADUATE INSTRUCTION, AND THE EXTENSION OF
THE ELECTIVE SYSTEM

To the Committee of Trustees of Columbia College on the Course and Statutes:

GENTLEMEN: The circumstances of the College are at length such as to make it possible to provide for giving instruction of a more advanced character than is required in the undergraduate classes. This is an object which many members of the Board of Trustees have long esteemed to be desirable, and of which the accomplishment has been regarded as only a question of time. Educational institutions of a nominally superior order are numerous enough in our country, but those which possess any just claim to be considered fountains of really high learning are as yet comparatively few. The country has, nevertheless, need of such institutions, as is made evident by the yearly increasing numbers of American youth who resort for the completion of their education to the Universities of the Old World. Such institutions will accordingly be erected, but it is hardly to be expected that they will be built from the ground up, on new and independent foundations. They will rather be developed out of existing institutions already well established, well endowed, and possessed of instrumentalities in actual operation, adequate, with some modification of the mode of their application, to accomplish the new and higher work proposed, without interfering with their present usefulness, or materially increasing the burden of their maintenance. The number of educational institutions in our country capable of such development is not at present great, nor is it important that it should be. It is far from being desirable, in the interest of the higher education itself, that universities (employing the term in the commonly accepted sense) should be numerous relatively either to the population or to the schools of lower grade. It is, on the other hand, vastly preferable that they should be few, strong, and largely attended, rather than many and, as an almost necessary consequence, ill-supported and feeble.

Circumstances already very distinctly indicate certain of the educational institutions of our country as destined inevitably to occupy this higher grade. There are others, doubtless, which will yet be added to the number, whose destiny is at present less clearly manifest; but as to these few there can be no possibility of mistake. Among the conditions which will aid in determining the future of these institutions is, of course, the present possession of adequate financial strength; but another, which is of nearly equal importance, is geographical position. Many reasons conspire to make the city of New York the fittest place on this continent for the growth of an educational institution of the highest order. It is not merely because of its superior population or its superior wealth that it possesses this advantage. It is because here are gradually concentrating themselves, in the most marked degree, all the most important accessories to mental cultures, all that most strikingly illustrates the triumphs of the human intellect in science, in letters, and in the arts of civilized life. And it is because in the ebb and flow of the vast human ocean which overspreads the continent, this city is the point toward which the currents of the population are continually tending, or from which they are retiring; and this is therefore, the point in which knowledge may be most conveniently sought, and from which it may be most easily diffused.

Columbia College occupies this very favorable position. It has already an honorable and well-earned reputation. It has around it a large body of Alumni interested in its prosperity, zealous for the promotion of its usefulness, and occupying, many of them, positions of influence in the social and political world. It has gathered together the most important of the instrumentalities necessary to the efficiency of instruction, or to the aid of literary research or scientific investigation. It has built up professional schools which, in there several specialties, take easily precedence of all others of their class in the United States. Its several faculties embrace men accomplished as scholars, thoroughly versed in the various departments of science, and profoundly learned in history, philosophy, and the law. What it needs to give it the character of a true university is to open its doors to aspirants for knowledge, who are seeking, not, as at present in the undergraduate department, knowledge in its mere rudiments, nor, as in the professional schools, knowledge for its immediately useful applications, but knowledge in the largest sense—knowledge pursued for its own sake, or to qualify the learner to contribute on his own part to the intellectual progress of the race.

To aspirants of this class opportunities may be offered for instruction in a great variety of subjects, without any addition, not demanded in the interest of the undergraduate department alone, to the present educational staff. To this end it is only necessary to adopt the simple expedient of distribution the hours of instruction over a larger portion of the day than they now occupy. According to the existing arrangement, all instruction is given within the limited space of three hours daily, and these are filled up with the exercises of the undergraduate classes. It is true, that within this limit it frequently happens that officers have unemployed hours; but these are not so distributed as to permit the arrangement of a working system of graduate instruction. To make such a system practicable, it is necessary that the subjects which a student may desire to combine should be taught at different hours. And this condition cannot be fulfilled without scattering the exercises over the greater part of the day. This is what is done in the School of Mines, in which all the hours from nine or ten o’clock in the morning till four or five in the afternoon are made available in one form or another for purposes of instruction.

But this expedient, while it provides that instruction shall be going on at every hour in one department or another, involves to the individual student the consequence that, between the hours in which he is engaged in class, there will occur other hours when he is without any scholastic exercise to employ him. During these hours it is proper that he should be occupied in study; and so he might be presumed to be were his residence in every instance within easy distance from the College. The institutions which provide lodgings for their students upon their own grounds find no difficulty in this matter. At Harvard University, for instance, exercises occur at every hour from nine till five; but the students, when not under instruction, are expected to study in their rooms. Since students with us have no private rooms, and are, in general, too distant from their homes to study there, the distribution of time proposed would hardly be judicious or just unless provision should be at the same time made for usefully employing the hours intervening between class duties. In the School of Mines abundant occupation for these hours is found in drawing, laboratory work, or practice in the use of tools and instruments. For the undergraduates it would be necessary to provide study rooms, to be used by students in common, by setting apart for the purpose some portion of the buildings not as yet otherwise assigned. The upper floor of the recently-erected building is entirely suitable to this object, and is quite sufficient. It is not needed for any present college uses, nor for any likely to arise so long as the old building continues to stand; and it is very evident that that building cannot be demolished until another shall have been erected somewhere else. Its demolition would necessitate the provision of other accommodations for the great variety of operations now going on in it; and though such might be found for the physical department in the new building, yet, for the Herbarium, for the classes in French, German, Mechanical Engineering, and Descriptive Geometry, for the operations of air and water analysis, for the library overflow, the gymnasium, etc., etc., no adequate space could be found there disposable. The future wants of the institution, for which the remaining floor of the new building is presumably held in reserve, will, therefore, undoubtedly be provided for in some manner not likely to interfere with any present disposition which may be made of that floor; and hence the devotion of that now unoccupied space to the uses of a study room cannot on that account be regarded as objectionable.

Supposing this arrangement to be adopted, it is easy, without adding to the number of our instructors, to plan a system of instruction for graduates which shall embrace reading or lecture courses in the Classics, English Literature, Anglo-Saxon, Philosophy and the History of Philosophy, Physiological Psychology, Political Science, Public Economy, History, the Higher Mathematics, Mechanics, Astronomy, Physics, Chemistry, Geology, and certain branches of Natural History, to occupy, on an average, not less than two hours weekly each. There can be no doubt that, when this system shall have been fully inaugurated, the number of students attracted by it will so far add to the revenues of the College as to justify the appointment of additional instructors on subjects not included in the above enumeration, such as Foreign Literature, Hebrew, Oriental Literature, Comparative Philology, Ethnology, Archæology, Natural History in its various ramifications, Animal and Vegetable Physiology, Methods of Research in Physics and Chemistry, Physical Astronomy, Architecture and the Fine Arts, and any others which may be necessary to make the institution a school of universal knowledge.

For the purpose of illustrating the practicability of carrying the proposed plan into effect without interfering with the undergraduate course already in operation, an ideal scheme of attendance is annexed to this communication, showing a distribution of the subjects of instruction to the hours of each day throughout the week, by means of which the convenience of instructors, as well of students, both graduate and undergraduate, may be satisfactorily provided for.

ENLARGEMENT OF THE ELECTIVE SYSTEM FOR UNDERGRADUATES.

An advantage incidental to the distribution of the hours of instruction over the entire day will be the opportunity it affords for giving to the students of the undergraduate department a greater liberty of option than they at present enjoy in the selection of their studies. It appears to the undersigned that the collegiate course for the student of our day cannot properly he made so severely a course of mere mental discipline as it was judicious to make it in the last century, or even during the earlier years of the present. The average age of our graduates is that of fully grown men. The age of the applicant for admission, a century ago, ranged from ten to fourteen years. The four years of college life embraced at that time the period of gradual mental as well as of physical development and growth; and it was eminently proper that the studies enforced upon the learner should be adapted rather to train the faculties than to inform the mind. At present this period is spent mainly in the preparatory schools, and it is in them that this work of mental gymnastics should be chiefly carried on. The end of college instruction is not merely to discipline, but also to inform; and since men differ no less in their mental than in their personal characteristics, it follows that different individuals do not acquire the same kind of knowledge with equal facility, nor are they equally profited by it. This provision of nature is one of great and beneficial importance to the progress of the race; for inasmuch as it is impossible that any one shall be proficient in all departments of human knowledge, the fact that each seeks spontaneously some special field insures the certainty that every such field will be energetically explored.

It seems, therefore, to be proper that at that period of comparative maturity at which young men begin to be conscious of their fitness for some particular pursuit or vocation, and desirous to acquire the knowledge most likely to be advantageous in following it, they should be allowed, to a pretty large extent, to select their subjects of study from among those which are in harmony with their tastes and aspirations. That public opinion in the social and the educational world recognizes this propriety is manifest in the flourishing condition of the institutions in which the elective system of study has been largely developed, and in the rapidity with which those institutions have grown in popular favor. In our own College the introduction of this system on an extended scale has been hitherto impracticable, for the same reasons which have made it impossible to provide for the instruction of graduates; yet the results of the limited trial of it which we have made have been altogether favorable.

The introduction of the elective system, however, in its largest latitude, does not by any means imply the necessity of discarding the system which at present exists. If there should be any who prefer still to cling to traditions, they may enter themselves from the beginning for the course of study prescribed for the degree of Bachelor of Arts in our statutes as they stand, and follow the same undeviatingly to the end. Degrees in Arts may be reserved to be conferred on such only; while those who prefer the optional course may be graduated as Bachelors of Letters or Bachelors of Science, according to the character of the studies which they chiefly pursue.

The adoption of a scheme of study largely elective would not necessarily require an increase in the number of instructors employed. As it is true, however, that, in such a system, the modern languages ought pretty fully to enter, it would be advisable to add to our present corps, a tutor in Italian and a tutor in Spanish. As to the French, arrangements could probably be made by which the instructor in that language in the School of Mines would teach also in the College; and for the German, we have already a professor who has hitherto given instruction in his proper department only under circumstances of serious embarrassment, and who would find the proposed change advantageous to his usefulness.

An ideal scheme of attendance accompanying this communication shows in what manner the exercises may be arranged so that students pursuing elective courses may attend along with candidates for degrees in Arts, so far as the subjects of study of these two classes of students are identical. Another appended paper shows the variety of particular courses of study which may be offered in the several departments indicated only by general titles in the scheme of attendance. In this latter paper are embraced some titles which do not fall within the special province of any of our present instructors; such as Hebrew Sanskrit, the Evidences of Religion, Anatomy, Physiology, and the principles of the Common Law.

In regard to the Evidences, there is no doubt that, under the voluntary system, the Trustees would think it advisable to reestablish the course; since with the abandonment of the principle of compulsion would disappear the reasons which have led to its discontinuance heretofore.

As to Anatomy and Physiology, arrangements could be made not involving any considerable expense, with Professors of the Medical School, to give annual courses adapted to the wants of the general learner, of great interest and value, though not extending to more than ten or twelve lectures each. A brief course of similar extent, upon the principles of the Common Law, could be given by our principal Law professor, embracing information highly important to educated men, but not now easily obtain able except in professional schools. Such a course used to be given to the Seniors at Yale College during the lifetime of the late Judge Daggett, which was always fully attended, though attendance was merely voluntary.

As to Sanskrit, a language occupying every year more and more the attention of scholars everywhere, there would be very little difficulty. Some of our own recent Alumni have already honorably distinguished themselves in this interesting study. One of our Fellows in Letters of the Class of 1878, now pursuing his studies at the University at Leipzig, had made himself by his own private and independent efforts a proficient in Sanskrit literature before his graduation, and since entering upon the course at Leipzig has received signal marks of approval and consideration from the professor in that department there. Another, of the Class of 1875, after completing his course of study abroad, has returned to this country bringing flattering testimonials to his attainments in general scholarship, and especially as to his proficiency in this particular branch. This young man, or some other similarly qualified, could be appointed to fill one of the new tutorships which it will presently become necessary to create in Greek or Latin in our College, and could be engaged at the same time to take charge of the instruction of any class which might be formed in Sanskrit. Considering that the condition of things is such in College at present as to make it impossible any longer to defer increasing the number of subordinate officers, it may be regarded as a happy concurrence of circumstances which enables us by the same act to provide a competent instructor in a subject in which as yet in this country the proficients are few.

All the subjects, however, for which instructors cannot be found among the members of our present Faculties, unless the modern languages be excepted, may be omitted, if necessary, from our scheme, till the success of the measure shall permit them to be provided for without adding to the burdens of the treasury.

PREPARATION OF YOUNG MEN FOR THE CIVIL SERVICE.

Since the passage of the resolution in regard to graduate classes, now before the Committee, a plan has been proposed for establishing, in addition to the special schools connected with the College already in existence, a new one of an original character, designed to meet what is believed to be a want of the present time, by preparing young men to engage intelligently in the service of the government. Though as yet in our country the civil and consular service has not been securely placed beyond the control of influences purely political, the popular sentiment manifests itself more and more strongly every year in favor of such a separation, and enough has been already accomplished to make it evident that, at least in the lower ranks of this service, proper qualification and the personal merit of the aspirant are likely hereafter to carry with them more weight in the competition for office than political favor. Even, moreover, should the so-called Civil Service reform receive a check, the course of instruction which it is proposed to give in the new school would still prove attractive to young men who desire to fit themselves to deal with questions of public interest, whether in professional life, or as politicians, or as managers of public journals. A school of a character similar to this has for several years been in successful operation in Paris, and counts among its teachers some of the most eminent publicists of France.

The proposition referred to proceeds from our Professor of Political Science, and is set forth in its details in a letter addressed by him to the undersigned, which is given below. As the proposed school involves the question of both graduate and undergraduate instruction, its consideration falls within the scope of the resolution referred to the Committee. An ideal scheme of attendance hereto appended shows in what manner the exercises of this course may be combined with those of the graduate and undergraduate classes, so that each may be accessible, if desired, to the students of the others.

In order that this project may be carried into effect, it would be necessary to appoint, within the next two years, two additional instructors, at salaries of $2,500 each. Within the same period the number of students entering for the course might reasonably be computed at not less than fifty, and would probably be greater. It would conduce very much to the success of the undertaking that the Law School should be removed from its present situation and united with the other departments upon the same ground, though that need not be regarded as in dispensable.

Following the letter of Professor Burgess below, will be found, marked A, a succinct résumé of the essential features of the plan herein recommended, and also of that of the proposed School of Preparation for the Civil Service, marked B. Also, marked C, an estimate of the probable effect upon revenue and expenditure of the adoption of these projects; and finally, an enumeration, marked D, of the subjects of the several courses of instruction which the proposed plan will embrace. The hypothetical schemes of daily attendance referred to above are presented separately.

All which is respectfully submitted,

F. A. P. BARNARD, President.

COLUMBIA COLLEGE, February 25, 1880.

__________________________

LETTER OF PROFESSOR BURGESS TO THE PRESIDENT.

323 West 57th St., Feb. 20th, 1880.

My Dear Dr. Barnard:

It seems to me evident that the time has now fairly arrived, both in the history of this nation and of this University, when a decisive step forward in the development of the political sciences is positively and specially demanded.

In the history of the nation it is so, not only because the Republic has now reached those mighty proportions demanding the finest training, as well as the finest talent, for the successful management of its affairs, but because the Government itself has recognized this fact, and in its Civil Service reforms, which, I think, are now fairly planted and destined, under the proper influences, to a noble growth, has opened the way for an honorable career to the young men of the nation in the governmental service, which may be successfully pursued by the best intelligence, skill, and fidelity, offering itself without any reference to political influence or patronage.

In the history of the University it is so, not only because it is the bounden duty of a university, worthy the name, to teach all that has been gathered by the world’s experience in this as well as all other departments of superior knowledge, and to add continually thereto, but because, also, of its metropolitan situation, which fits it better than any other in the nation, both to place its students in immediate connection with the Civil Service examinations, so far as they now exist, and to exert its influence with greatest efficacy for the extension of the same throughout every branch of that service, as the indispensable condition of appointment to governmental office, and because I think I may assert that the foundation is now already fairly laid in our University for the development which I now propose.

The elements of the plan which I would suggest are, in outline, as follows:

I

The course of study shall extend over a period of three years, and comprise the following subjects:

FIRST YEAR.— The History of Philosophy; The History of the Literature of the Political Sciences; The General Constitutional History of Europe; The Special Constitutional History of England and of the United States.

SECOND YEAR.— The Roman Law—the general principles of the jurisprudence of the existing codes derived therefrom; The Comparative Constitutional Law of the Principal European States and of the United States; The Comparative Constitutional Law of the Different Commonwealths of the American Union.

THIRD YEAR.— History of Diplomacy; International Law; Systems of Administration, both of the Nation and the Commonwealths in the American Union; The Comparison of the Administrative System of the United States with those of the principal European States; Political Economy in all its Branches, and Statistics.

II.

The requirements for admission to this department shall be:

FIRST.— The completion, successfully, of the Junior Year in any regular college of the United States; or—

SECOND.—The passing of a successful examination upon all  the studies of the first three years of the Academic Department of this University; and—

THIRDLY. — Including therein, in either case, a fair reading knowledge of the French and German languages.

III.

The degree conferred, upon the completion of all the courses and after successful examination therein, shall be that of A.M., Ph.D.; and if, in addition to the courses in this department, the student shall have received the degree of LL.B., either from the Law School of this University or from any other School of Municipal Law in good standing, the degree shall be that of D.C.L. For the students of a single year’s standing the degree will be that of A.B.

IV.

The fee for instruction in this department shall be the same in amount as is paid by undergraduates. But members of the Senior Class in the Academic Department of the University may attend the courses of the first year, and members of the School of Law may attend any or all of the courses free.

Finally, my dear Doctor, I would add that, in my opinion, this plan can be now realized without much, if any, additional expense to the Treasury of the University. When in complete running order it will require the appointment of two assistants to the Professor now in charge of the department — one for the courses in Roman Law and its branches, the other for the courses upon the administrative systems of Europe and the United States. I am confident that two fit persons, both graduates of the Law Department of this University, and now about finishing their courses of special study upon these topics in European universities — the one at Berlin and the other at the Ecole Libre des Sciences Politiques in Paris–can be secured at salaries of $2,500 each per annum; and if one of these should be appointed, his office and salary to take effect from October 1st, 1881, and the other from October 1st, 1882, I fully believe that the receipts from the fees of students in the department would be sufficient to cover one, if not both, of these salaries, and in five years from this time would be a source of revenue to the Treasury. I am informed that in the city of New York  alone fifty or more vacancies occur every year in the Civil Service of the United States, from death, resignation, or inability to serve; and that, through the present method of Civil Service examinations, fifty or more appointments are therefore annually made to offices having salaries of from $1,200 to $5,000; and I feel assured that the institution of learning which shall first seize the opportunity for establishing among its departments a school of training for the Civil Service of the National Government, will not only increase its own prestige and be a national benefactor, but will also reap no mean financial reward therefrom.

Trusting that these suggestions may meet your favor and support,

I am, as ever, your most obedient servant,

JOHN W. BURGESS.

President F. A. P. BARNARD, LL.D.

*  *  *  *  *  *  *  *  *  *  *  *  *  *

(A.)

PLAN FOR THE INSTRUCTION OF GRADUATE CLASSES, AND FOR THE EXTENSION OR THE ELECTIVE SYSTEM IN THE UNDERGRADUATE DEPARTMENT.

1.—Scholastic exercises to be distributed over all the hours of the day from 9½ A.M. till 4 P.M. from Monday to Friday inclusive, and from 9½ till 11 on Saturday.

2.—Every student to be under instruction in class for three hours each day till Saturday, and for two hours on Saturday.

3.—Students, when not under class instruction, to be occupied in study in rooms set apart for the purpose.

4.—A recess of half an hour to be allowed at mid-day for lunch.

DEGREES.

1.—If no modification of the course of instruction or no enlargement of its scope be made, the only degree conferred at the close of the four years’ course to be the degree of Bachelor of Arts.

2.—Should the course be enlarged and liberty of option be increased, the degrees of Bachelor of Letters and Bachelor of Science to be also given.

3.—The course leading to the degree in Arts to remain unchanged, consisting as at present of a prescribed curriculum up to the close of the Junior year, and of a partially elective course during the Senior.

4.—For degrees in Letters and Science, the course to be identical with the course in Arts during the Freshman year, and to embrace as obligatory during the subsequent years all the subjects of that course in the departments of English Literature, History, and Political Science, occupying six hours in the Sophomore year per week; five hours in the Junior, and four hours in the Senior. The remaining hours up to seventeen per week to be occupied by elective studies. The degree in Letters to be conferred on those who choose as electives the Languages, Ancient or Modern, Literature, Psychology, and Philosophy; and the degree in Science on those who choose principally the Mathematics, Physics, Mechanics, Astronomy, and Chemistry. Specific regulations in regard to this subject to be made by the Faculty, with the approval of the Trustees.

INSTRUCTION TO GRADUATES

1.—Instruction to be given to graduates of this and of other Colleges who desire to fit themselves for literary or scientific work not professional; as for authorship, historical or philological research, scientific investigation, etc.

2.—Special courses of instruction adapted to the wants of such graduates to be arranged in the departments of Greek, Latin, Mathematics, Physics, Astronomy, Chemistry, Philosophy, Political Science, Natural History, and the Modern Languages. Only such courses to be attempted at present as can be conducted without materially increasing the staff of instructors.

3.—Notice to be publicly given before the close of the present academic year of the purpose to institute courses for graduates, with specification of such as may be commenced in October, 1880.

*  *  *  *  *  *  *  *  *  *  *  *  *  *

(B.)

SCHOOL OF PREPARATION FOR THE CIVIL SERVICE, DIPLOMACY, AND THE EDITORIAL PROFESSION.

1.—A School to be instituted with a definitely prescribed curriculum of instruction, designed to prepare young men for public life, whether in the Civil Service at home or abroad, or in

the legislatures of the States or of the nation; and also to fit young men for the duties and responsibilities of public journalists.

2.—Students of our own College, or of other Colleges in good standing, who have completed with credit the undergraduate course up to the close of the Junior year, to be allowed to enter themselves as students in this School.

3.—The course of instruction to extend over three years.

4.—At the close of the first year, the student, on passing an approved examination, to be entitled to the degree of Bachelor of Philosophy.

5.—At the end of the course, the student who passes all his examinations with approval, to be recommended to the Trustees for the degree of Doctor of Philosophy.

6.—Students of this School to be permitted to attend such exercises of the Graduate or Undergraduate department as their time will allow, the approval of the Professors in the School being first obtained.

*  *  *  *  *  *  *  *  *  *  *  *  *  *

(C.)

PROBABLE EFFECT ON REVENUE AND EXPENDITURE OF THE ADOPTION OF THE PROPOSED SCHEME.

I.—EXPENDITURE.

It must, in the first place, be considered that the exigencies of the College will require, whether this plan be adopted or not, the appointment, in anticipation of the opening of the ensuing academic year, of an additional tutor in Greek, of one in Latin, and of one also in Mathematics. This will involve an increase of annual expenditure not to be charged to the present project, of $3.600.

For the supervision of students in their study rooms it will be expedient to appoint two Proctors at salaries of one thousand dollars each per annum. It is probable that this amount might be reduced by arranging that Tutors may act as Proctors.

The department of Modern Languages, in regard to which our College is now deplorably deficient, would require the appointment of an instructor in Italian, and an instructor in Spanish, who might probably be engaged for about $1,300 per annum each. The instructor in French of the School of Mines would probably be willing, for a slight increase of compensation, to take charge of the French classes in College also.

These comprise all the additions to our annual outlay which it seems to be necessary at present to make. Summed up they are as follows:

Two Superintendents of study rooms $2,000
Two Instructors in Modern Languages $3,000
Increase of salary of Instructor in French $500
Total $5,500

II.—REVENUE.

The tuition fee now charged in the Undergraduate Department is less by fifty dollars per annum than that which is charged at Harvard College or at the University of Pennsylvania. It seems fitting that, in so largely increasing the advantages offered to students here, we should make our charges equal at least to what they are elsewhere. To increase the fee from $100 to $150 per annum would not diminish the number of our undergraduate students, and hence, even should the new attractions fail to add to our numbers, the effect of this measure would be to add $10,000 to our annual revenue.

It may, however, be safely estimated that, before two years shall have passed under the new system, we shall have at least twenty graduate students, and that the number of undergraduates will be more than equally increased. The following, then, are the items of the probable increase of revenue, viz.:

From increase of tuition fee $10,000
From graduate students (say 20) $3,000
From increased number of undergraduates (say 20 also) $3,000
Total $16,000

The probability, therefore, is that the adoption of the plan, if accompanied as it should be with a simultaneous increase of the tuition fee, will tend to enlarge rather than diminish the annual balance in the treasury.

If, now, we consider the further effect of the establishment of the proposed new school of preparation for the Civil Service, we shall have to add to the annual outlay the amount of the salaries of two Instructors or Adjunct Professors, one in Roman Law and one in Administrative Law, each of $2,500. On the other hand, we may safely count on attracting to this school, in the course of the next two or three years, at least fifty students—a number likely afterward largely and rapidly to increase. We may assume, then, at the end of the second year, something like the following, viz.:

I.—EXPENDITURE.

Total as above $5,500
Salaries of two instructors $5,000
Total $10,500

 

II. REVENUE.

Total as above $16,000
Fees of fifty students $7,500
Total $23,500

Leaving a probable balance in favor of the treasury of $13,000 per annum.

*  *  *  *  *  *  *  *  *  *  *  *  *  *

(D.)

COURSES OF STUDY.

The subjects or authors herein enumerated as for undergraduates, are in general those taught during the academic year 1878-9. In the classical departments these are liable to be changed from year to year. Should the proposed plan be adopted, every course not marked as obligatory will be open for election to all undergraduates above the Freshman grade, without regard to year.

DEPARTMENT OF GREEK.

1.— Iliad or Odyssey, one term 3 hours per week
Freshmen,
Obligatory.
2.— Herodotus, one term
3.—The Odyssey 2 hours per week,
Freshmen,
Voluntary.
4.—Aristophanes and minor poets 2 hours,
Sophomores,
Voluntary.
5.—Euripides, one term 3 hours per week,
Sophomores,
Elective.
6.—The Memorabilia, one term
7.—Sophocles, one term 3 hours per week,
Juniors,
Elective
8.—Plato, one term
9.—Æschylus, one term 3 hours per week,
Seniors,
Elective
10.—Demosthenes, one term
11.—(To be announced annually in advance) 2 hours per week,
Graduates.
12.—(To be announced annually in advance) 1 hour per week,
Graduates.

 

DEPARTMENT OF LATIN.

1.—Pliny 3 hours per week,
Freshmen,
Obligatory.
2.—Horace, one term 3 hours per week,
Sophomores,
Elective.
3.—Livy, one term
4.—Juvenal, one term 3 hours per week,
Juniors,
Elective
5.—Cicero De Officiis, one term
6.—Terence, one term 2 hours per week,
Seniors,
Elective
7.—Catullus, one term
8.—(To be announced annually in advance) 2 hours per week,
Graduates,
Elective.
9.—(To be announced annually in advance) 1 hour per week,
Graduates,
Elective.

 

DEPARTMENT OF MATHEMATICS.

1.—Geometry, one term 5 hours per week,
Freshmen,
Obligatory.
2.—Algebra, one term
3.—Curves of Second Order, one term 3 hours per week,
Sophomores,
Elective.
4.—Trigonometry, Mensuration and Surveying, one term
5.—Differential and Integral Calculus 2 hours per week,
Seniors,
Elective
6.—Calculus of Variations 2 hours per week,
Graduates.
7.—Quaternions 2 hours per week,
Graduates.

 

DEPARTMENT OF MECHANICS AND ASTRONOMY.

1—Analytical Geometry, one term 3 hours per week,
Juniors,
Elective.
2.—Mechanics, one term
3.—Astronomy 3 hours per week,
Seniors,
Elective.
4.—Practical Astronomy 2 hours per week,
Graduates.
5.—Spherical Astronomy 2 hours per week,
Graduates.
6.—Analytic Mechanics 2 hours per week,
Graduates.
7.—Physical Astronomy 2 hours per week,
Graduates.

 

DEPARTMENT OF PHYSICS.

1.—Elementary Physics 2 hours per week,
Freshmen,
Obligatory.
2.—Heat and Electricity 2 hours per week,
Juniors,
Elective.
3.—Optics and Acoustics 3 hours per week,
Seniors,
Elective.
4.—Mathematical Physics 2 hours per week,
Graduates.
5.—Methods of Physical Research 2 hours per week,
Graduates.

 

DEPARTMENT OF CHEMISTRY.

1.—Elementary Chemistry 1 hour per week,
Freshmen,
Obligatory.
2.—Theoretic Chemistry 2 hours per week,
Sophomores or Seniors,
Elective.
3.—Applied Chemistry 2 hours per week,
Juniors,
Elective.
4.—Organic Chemistry 2 hours per week,
Graduates.
5.—Methods of Chemical Research 2 hours per week,
Graduates.

 

DEPARTMENT OF PHILOSOPHY.

1.—Logic 2 hours per week,
Juniors,
Obligatory.
2.—Psychology 2 hours per week,
Seniors,
Obligatory.
3.—Philosophy 3 hours per week,
Seniors,
Elective.
4.—Physiological Psychology 1 hour per week,
Graduates.
5.—History of Philosophy 2 hours per week,
Graduates.

 

DEPARTMENT OF ENGLISH LITERATURE.

1.—Rhetoric 3 hours per week,
Freshmen,
Obligatory.
2.—History of Literature 2 hours per week,
Sophomores,
Obligatory.
3.—English Classics 1 hour per week,
Sophomores,
Obligatory.
4.—Logic 2 hours per week,
Juniors,
Obligatory.
5.—English Classics 1 hour per week,
Juniors,
Obligatory.
6.—The Early English Authors 2 hours per week,
Graduates.

 

DEPARTMENT OF THE MODERN LANGUAGES.

1a.— French, Elementary Course 3 hours per week each,
Undergraduates.
1b.—Course of French Literature
2a.— German, Elementary Course 3 hours per week each,
Undergraduates.
2b.— Course of German Literature
3a.— Italian, Elementary Course 3 hours per week each,
Undergraduates.
3b.— Italian Literature 1 hour per week each,
Graduates or Undergraduates.
4a.— Spanish, Elementary Course 3 hours per week each,
Undergraduates.
4b.— Spanish Literature 1 hour per week each,
Graduates or Undergraduates.

 

DEPARTMENT OF HISTORY AND POLITICAL SCIENCE.

1.—Outlines of General History 3 hours per week,
Sophomores,
Obligatory.
2.—History of England and the United States, one term 3 hours per week,
Juniors,
Obligatory.
—Political Economy, one term
3.—Political Economy and Statistics 3 hours per week,
Seniors,
Obligatory,and School of Civil Service.
4.—British and American Constitutional history 2 hours per week,
Seniors,
Obligatory.
5.—General Constitutional History 3 hours per week,

School of Civil Service.

6.—Literature of Political Science 1 hours per week,
3 hours per week,
School of Civil Service.
—Comparison of Constitutional law of Europe and the United States
7.—Systems of Administration 3 hours per week,
2 hours per week,
School of Civil Service.
—Comparison of Constitutional Differences of the American States
8.—Comparison of Systems of Administration 5 hours per week,
School of Civil Service.

 

ROMAN AND INTERNATIONAL LAW.

1.—Roman Law 3 hours per week,
School of Civil Service.
2.—International Law 3 hours per week,

School of Civil Service.

 

DEPARTMENT OF GEOLOGY.

1.— General Geology 1 hour per week,
Seniors,
Elective.
2.— Palæontology 1 hour per week,
Seniors,
Elective.
3.— Lithological Geology, first term 3 hours per week,
with School of Mines.
4.— Cosmical and Historical Geology, second term 3 hours per week,
with School of Mines.
5.— Economical Geology 2 hours per week, through the year
with School of Mines.

 

NATURAL HISTORY.

Botany, first term 2 hours per week,
with School of Mines.
Zoology 1 hour per week, throughout the year
with School of Mines.
Vegetable Physiology, Animal Physiology, Anatomy—Human and Comparative To be provided for.
Mineraology and Crystallography 2 hours per week,
with School of Mines.

 

LAW.

1.— Outlines of British Common Law Ten lectures.

 

SEMITIC AND ORIENTAL LANGUAGES.

Hebrew 2 hours per week,
To be provided for.
Sanskrit 2 hours per week,
To be provided for.

 

 

Source: Columbia University Archives. Historical Subject Files, Series I: Academics and Research/Series VIII: Events/I. Box 289. Folder 1 “Political Science, Faculty of, 1920s-1930s”.

Image Source: John W. Burgess, from the Columbia University, Department of History webpage: A Short History of the Department of History.

 

Categories
Economics Programs Harvard Teaching

Harvard. Haberler and Chamberlin fight over last-minute course changes, 1942-43

Exogenous shocks are really useful for finding out how the economy works. They also help dear colleagues reveal themselves when their private interests conflict with those of other colleagues in particular or with departmental needs in general. The U.S. entry into the Second World War forced several adjustments in the graduate and undergraduate instructional staffing at the Harvard economics department.

This post provides some light on the time Gottfried Haberler was asked to teach the first of the two term graduate economic theory sequence for the academic year 1942-43. The course was a direct descendent of Frank Taussig’s Economics 11 (the expansion of course offerings over the decades required moving into 3 digits for some course numbers and a zero was dropped into the middle of “Economics 11” to obtain “Economics 101”). At the last minute Chairman Edward Chamberlin decided that he wanted “his” course back for both semesters but Gottfried Haberler was clearly not one to go quietly. And so we witness the performance of an academic drama before colleagues, of Professor X and Professor Y claiming conflicting rights to a particular course.

The record presented here is incomplete. I have been unable to find Haberler’s written plea on his own behalf. Reading the material one might think that Chamberlin got his way and Haberler was left to find another course to satisfy his annual teaching obligation. However, a look into the annual report of the President of Harvard College for 1942-43 finds that as far as the staffing of Economics 101 in 1942-43 goes, ex ante equals ex post. The course was ultimately divided that year between Messrs. Haberler and Chamberlin.

___________________________

Economics 101: syllabi (with links to most readings) and examinations for fall and spring terms 1941-42 taught by Edward Chamberlin.

___________________________

Who ended up teaching what 1942-43

Edward Chamberlin

Economics 1a. First term, undergraduate course “Economic Theory”.

Economics 102b. Second term, graduate course “Monopolistic Competition and Allied Problems”.

Co-taught Economics 101 with Gottfried Haberler. Full-year graduate course “Economic Theory”. Presumably Haberler taught the first term and Chamberlin taught the second term.

Gottfried Haberler

Economics 18b. Second term, undergraduate course on the Economic Aspects of War,

Co-taught Economics 45a with Alvin Hansen. First term, undergraduate course  “Business Cycles”.

Economics 144. Graduate School of Public Administration Seminar “International Economic Relations”.

Co-taught Economics 101 with Edward Chamberlin. Full-year graduate course “Economic Theory”. Presumably Haberler taught the first term and Chamberlin taught the second term.

___________________________

Ex Ante Course Announcement

Economics 101. Economic Theory

Mon., Wed., and (at the pleasure of the instructors) Fri., at 12. Professors Chamberlin and Haberler.

Source: Final Announcement of the Courses of Instruction offered by the Faculty of Arts and Sciences during 1942-43. Official Register of Harvard University, Vol. XXXIX, No. 53 (September 23, 1942), p. 55.

___________________________

Ex Post Course Enrollment and Staffing

[Economics] 101. Professors Chamberlin and Haberler. — Economic Theory.

Total 24: 16 Graduates, 4 School of Public Administration, 1 Graduate Business School, 3 Radcliffe.

Source: Harvard University. Report of the President of Harvard College 1942-1943, p. 47.

___________________________

Presumably the statement prepared by Edward Chamberlin (referring to himself in the third person)

October 9, 1942

Course Economics 101 was announced in the catalogue for 1942-3 to be given jointly by Messrs. Chamberlin and Haberler. This arrangement was never considered as final but was subject to adjustment at the beginning of the college year in view of the general uncertainty as to the status of such of the graduate instruction until enrolments in various courses were known. (In particular, it seemed likely that either 102b or 163 or both might be bracketed, thus freeing either one half or one full course of Mr. Chamberlin’s time). It was, however, agreed between Mr. Haberler and Mr. Chamberlin that, in case the course were given jointly, Mr. Haberler would give the first half year and Mr. Chamberlin the second. Several times prior to the opening of college Mr. Haberler asked Mr. Chamberlin about the status of the course and was told that unfortunately nothing final could be decided until enrolments were known; it was agreed, however, that Mr. Haberler would take the first meeting, or meetings, of the course until a decision was reached. The matter was mentioned on Friday morning, October 2, at a casual meeting between classes at which time, since no final decision had been taken, Mr. Chamberlin said that it was still possible that the arrangement might stand. On Saturday, October 3, a final decision to take back the course was communicated to Mr. Haberler after considering a number of factors, among which were the following:

  1. The enrolment in Economics 102b was only two, plus five auditors. This course had always been given in the second semester, thereby opening it to the first year students who had had the first semester of 101. The bracketing of 163 made it possible to revert to this disposition of 102b, (or to bracketing it later on if this seemed necessary). The chief obstacle to Mr. Chamberlin’s giving the first half and therefore all of 101 was thereby removed.
  2. The class list of 101, received Friday afternoon, revealed that of 16 [or 18?] student then enrolled in the course all but two were foreigners. Many of these would have serious problems of adaptation to academic work in a new language and in a new country, and it seemed for the reason especially desirable to unify the introductory course in theory under one direction during the current year.
  3. During the past, two years the course had, for better or worse, been split both vertically and horizontally, not by action of the department but on the initiative of Mr. Chamberlin. This was done in part to open greater possibilities for discussion through smaller sections, and in part to share the course with others who wanted very much to teach theory. At no time during that period had Mr. Chamberlin given less than a full year of the course, and its outline and organization had always been his. It was his sincere belief that now that the course was again of manageable size the department would wish it to be given as it had directed earlier, and that he was fully competent to make the decision. At that time the work of the year had not really begun.

However, Mr. Haberler objected so strongly to the change that in order to settle the matter amicably, Mr. Chamberlin proposed on Sunday afternoon, and Mr. Haberler agreed, that the matter be left to a committee composed of Professor Crum as Chairman and other members to be chosen by Professor Crum. As this Committee could not possibly be assembled and give a decision before the Monday meeting of the course it was agreed that Mr. Haberler would take that meeting and that the Committee shouId render a decision before the Wednesday meeting. The decision was in fact rendered Tuesday afternoon and was unanimous that Mr. ChamberIin should give the course, When apprized of this decision, Mr. Haberler said he would like time to consider whether or not he was willing to accept it. From this point on Mr. Chamberlin became a passive duopolist, leaving all initiative to Mr. Haberler, who proposed that he take the Wednesday meeting of the course, finishing matters which he had begun on Monday, give a cut on Friday (there was a holiday the following Monday), and decide sone time before the Department meeting whether or not he would like to bring the matter before the Department. Meanwhile, the Committee had decided that certain questions which it had discussed apart from the immediate issue should be brought before the Department at its meeting October 13th. Mr. Haberler’s final decision on Thursday was that if the Department is going into the whole theory question anyway, they should also decide on the present status of Economics 101.

___________________________

Chairman Chamberlin announces his attention to return statement with statement

October 10, 1942

Dear Leonard [Crum]:

It has occurred to me that, since Haberler has given you in writing a statement of the facts as he sees them, I might, even at this late date do the same. My own statement will add some details and perhaps present a different emphasis at one or two points It may be used at your discretion in whatever way you think best, (including, of course, the possibility of no use at all beyond your own reading). I am sending a copy to Haberler.

Sincerely,

E. H. Chamberlin

___________________________

Chairman Chamberlin makes his written statement available to the department

October 17, 1942

To Members of the Department of Economics:

In view of the fact that Professor Haberler’s statement with respect to Economics 101 had some circulation prior to last Tuesday’s meeting and was also read in the meeting, whereas my own statement has to this moment been seen only by Professor Crum and one other member of the Department, I should like now to make both equally available to any who may wish to consult them. Accordingly, they will both be found in the blue folder in Mrs. Arnold’s office. Also in the blue folder are: (1) The minutes of the last three meetings, and 2) The report of the Chairman to the Dean of the Faculty covering the work or the Department for the past year.

Chairman [Chamberlin]

___________________________

Special Committee sides with Chamberlin

CONFIDENTIAL

(for use of Department
of Economics officers
only)

Report of a Special Committee
on the assignment for teaching Economics 101.

13 October, 1942

On Monday, October 5, the Chairman of the Department brought to my notice a personal disagreement between himself and Mr. Haberler concerning the assignment for teaching Economics 101, and asked that I serve as Chairman of a Special Committee to “arbitrate” in the case, and report before the meeting of Economics 101 on Wednesday the 7th. I was instructed to associate with myself such members of the Department as I saw fit in making up the Committee. I asked Mr. Burbank, formerly Chairmen of the Department, to be a member, and also four other members of the Department who have at present no active part in the teaching of economic theory and whose views on the matter at issue were unknown to me. One of these individuals was unable to serve because of his inability to meet with the Committee at any time available for meeting within the interval during which action had to be taken. The Committee, therefore, was made up as follows: Crum, Chairman, Black, Burbank, Dunlop, and Usher.

The Committee met and considered to the best of its ability all aspects of the case, and herewith reports certain recommendations to the Department for such action as it wishes to take. The Chairman of the Committee reported on Tuesday afternoon the 6th to Messrs. Chamberlin and Haberler the findings of the Committee in outline form because the Chairman thought that the two individuals concerned might have agreed to abide by the finding of the “arbitration” and might be willing to put the findings into effect immediately. The Chairman of the Committee did, however, report to both participants in the controversy that he did not regard the Committee as being clothed with any conclusive authority and that unless the participants in the controversy both accepted the findings of the Committee those findings would have to go to the Department as recommendations and would be subject to such action as the Department saw fit to take.

Statement of the issues.

Course Economics 101 is announced in the spring issue of the current Courses of Instruction pamphlet as to be given jointly Messrs. Chamberlin and Haberler, and I am informed that they had during the summer agreed among themselves that, in case the course was given jointly, Mr. Haberler would give the course during the first half year and Mr. Chamberlin during the second. Late in the week in which instruction of the present half year began Mr. Chamberlin indicated to Mr. Haberler that he thought the entire course should be given by Mr. Chamberlin. Mr. Haberler objected to any such change and insisted that he continue to give the course during the first half year. The issue, accordingly, was whether the conduct of the course should go forward on the basis of joint responsibility of Mr. Haberler in the first half year and Mr. Chamberlin in the second half year, or should be restored to the basis prevailing for several years in which Chamberlin gave the full course.

History of the case.

After the retirement of Professor Taussig, Course 101 (formerly called Course 11) was given for several years by Mr. Schumpeter by an arrangement which was understood to be provisional and subject to later change. At the end of this interim, after extended consideration of the needs and purposes of the Department with respect to the teaching of the several courses in economic theory, the Department took specific action directing Mr. Chamberlin to teach Course 101. At the same time arrangements were agreed upon by which several other specialists in economic theory in the staff of the Department participated in the instruction in economic theory. Presently the enrolmont in Course 101 became so large that its conduct as a single course by the discussion method became difficult; and, without specific vote of the Department, the course was divided into two sections, with one conducted by Mr. Chamberlin and Mr. Taylor in 1940-41 and by Mr. Chamberlin and Mr. Leontief in 1941-42, the other by Mr. Haberler and Mr. Chamberlin in both years.

With the decline in enrolment which has occurred, no occasion for such splitting of the course persists, and it has long been forseen that Economics 101 would be conducted as a single section during the present year. In recognition of this, an arrangement was made, without specific action by Department vote, to announce Economics 101 for the present year as to be given jointly by Messrs. Chamberlin and Haberler. At the time this arrangement was made the expectation was that Mr. Chamberlin would be giving during the first half year Economics 1a and Economics 102b and that he should not be called upon to carry the additional instruction involved in teaching Economics 101 during the first term.

The initial enrolment in Economics 102b was so small that the course has been withdrawn from the first term offering, and although it is announced for the second term doubt remains whether the enrolment will be sufficient even then to warrant giving it. In recent months, various changes in personnel of the Department and the necessity of distributing the teaching and other load in all branches of the Department work as fairly and efficiently as possible have resulted in various changes in the assignments of particular officers to particular duties. In these circumstances it became possible for Mr. Chamberlin to resume during the first term instruction in Economics 101 without making his course load excessive.

In connection with the controversy, the Chairman of the Committee had a conversation with Mr. Chamberlin in which the latter presented his own views concerning the history of the case and the points at issue. Mr. Haberler submitted a written statement to the Chairman of the Committee setting forth his ideas on the matter. Those items were brought to the attention of the Committee by its Chairman. Following the meeting of the Committee, Mr. Chamberlin also submitted a written statement to the Chairman of the Committee. Either or both of the written statements will be laid before the Department on request.

Meeting of the Committee.

The Committee met on Tuesday, October 6. The Chairman gave the Committee a history of the case and a summary of the information available bearing upon the point at issue. The Chairman also informed the Committee that he did not understand that the Committee had any conclusive powers and would be obliged to report its findings in the form of recommendations to the Department.

The Chairmen specifically urged the Committee, therefore, in proceeding toward its findings to consider the wisdom of bringing in findings which, in its opinion, would probably be supported by the Department. The Chairman reminded the Committee that adequate treatment of the particular matter at issue might well involve (a) recommendations by the Committee concerning certain related matters affecting other courses; and (b) recommendations by the committee concerning certain longer run matters relating to the general question of our offering in economic theory. The Chairman discussed with the Committee certain basic principles bearing upon the case, and received the concurrence of all the members of the Committee in these principles. They are outlined below.

The Committee then proceeded to discuss the matter at issue and various related matters. Discussion by the members of the Committee was free and active and the Chairman made a special effort to call forth the views of each member of the Committee. After this discussion the Committee agreed upon a set of recommendations to be made to the Department, and to be reported to Messrs. Chamberlin and Haberler in the hope they would accept the findings. The agreement of the Committee was unanimous. Those recommendations are presented below.

Basic principles.

In approaching a set of findings with respect to the issues raised the Committee had in mind a series of basic principles in which members of the Committee concurred. Those are as follows:

(a) Because of its compressed personnel in wartime and because of the extraordinary wartime adjustments needed in its work: the Department has a peculiarly difficult task of assigning functions to its various officers with a view to getting the essential work of the Department done with such distribution of the burden as will be primarily efficient from the point of view of the Department and secondarily fair from the point of view of the individuals.

(b) Even in peacetime the needs of the Department and the objective of securing maximum efficiency in the performance of Department work transcend the interests and preferences of individual officers. Although in peacetime many concessions can be made with a view to accommodating the preferences and interests of individual officers and with a view to protecting the rights or supposed rights of individual officers, the Department would in general not recognize that such individual interests can overrule the general interest of the Department. In wartime this condition is even more emphatically true, and in such time the individual preferences and interests may be obliged to give way to the general interest of the Department more frequently and more extensively than in normal times. Throughout the duration of the war many if not all of the officers of this Department will be doing work which they prefer not to do and will be denied the opportunity to do work which they would like to do. Without such sacrifices the essential work of the University cannot be effectively handled in wartime.

(c) The Department and the University cannot afford to allow the general interest to be sacrificed because of informal commitments or quasi agreements made among individual officers when such agreements fail to take adequate account of the general interest of the Department, even though those who made the agreements acted in good faith. That agreements thus made may from time to time have to be set aside in the interest of the Department, and that such setting aside may involve some sacrifice by one or more individuals involved must be accepted as one of the costs of giving primary importance to the general interest of the Department. Ordinarily it is to be expected that individuals will refrain from making arrangements for which they have no power under the law of the Department; but ever if such arrangements are entered into under a grant of power, the individuals concerned must recognize that the Department itself has a clear right to final determination at one of its meetings.

(d) To the best knowledge of the Committee, the purpose of the Department with respect to the assignment of instruction in Course 101 remains as it was last officially determined by Department vote several years ago, namely, that Course 101 should be given by Mr. Chamberlin.

(e) Under the stress of war the Department may be obliged to sacrifice in part some branches of its work, and the Committee believes that graduate instruction will probably need to be sacrificed before instruction in undergraduate courses, tutorial instruction, and other Department work directed toward the teaching of undergraduates. A policy which exposes graduate instruction to the principal sacrifices is also likely to result in the most frequent disregard of personal preferences and even of supposed rights of individual officers; but presumably the Department would nevertheless feel that such a policy must be adopted and maintained.

Recommendations to the Department.

After considering the facts laid before it in connection with the matters at issue and in the light of its own agreement on basic principles, the Committee agreed unanimously to present the following recommendations to the Department at its meeting on Tuesday, October 13, and to report these recommendations at once to the parties in the controversy:

(a) That during the present year Mr. Chamberlin be assigned to conduct the entire Course 101.

(b) That, in view of chancing conditions which may mean that the Department’s present total offering in economic theory covering the entire range of courses in that field does not most satisfactorily meet the needs of instruction in that field, the Department promptly and earnestly reconsider the total offering with a view to making such changes as may be necessary in the next announcement of courses. The Committee makes no recommendation as to how the reconsideration should be conducted, whether by the appointment of a committee or by general Department discussion or by a combination. It also makes no specific recommendation as to any changes in the present offerings of courses, but merely notices that such a general reconsideration may well cover the possibility that Mr. Haberler might be asked to give work in economic theory.

(c) The Committee recommends that the Department consider asking Mr. Haberler to take charge of an additional half course during the present academic year, with a view to replacing the first half of Course 101 in rounding out his teaching assignment. The Committee specifically recommends that Course 18b be considered as one of the possibilities for additional instruction by Mr. Haberler; and makes this recommendation because on the one hand the Committee feels that the hurried arrangement by which that course was assigned jointly to four officers won Mr. Harris withdrew may have been ill-advised in that use of too numerous instructors in such a course may damage the continuity from the point of view of the student; and on the other hand the Committee believes that Mr. Haberler’s areas of specialization would enable him to handle this particular course very effectively.

(d) The Committee recommends that the Department consider carefully the question whether in determining that the enrolment in a course is so small that the course should be withdraw only those enrolled for credit should be counted, or whether in addition the auditors should be counted (this question was raised before the Committee in connection with Course 102b in which the first term enrolment was two members for credit plus five others. Course 102b has been withdrawn from the first term offering, but will be announced again for the second term, and the question posed above may at that time again be raised).

W. L. Crumm

___________________________

Thus spake the Dean

HARVARD UNIVERSITY
CAMBRIDGE, MASSACHUSETTS

FACULTY OF ARTS AND SCIENCES

Paul Herman Buck, Dean
Henry Chauncey, Assistant to the Dean
Jeffries Wyman, Jr. Assistant Dean

5 University Hall

January 12, 1943

To the Senior Members of the Department of Economics:

After considerable contemplation of the issue which has arisen between Professors Chamberlin and Haberler and which I have undertaken to arbitrate, I find I am in complete accord with the Report of a Special Committee on the assignment for teaching Economics 101, dated October 13, 1942. I commend especially as sound, the basic principles outlined on page 3 of that report and I accept as my official decision the recommendations to the Department given on page 4 of that report.

Frankly, it seems to me most unfortunate that the issue should have descended into personalities. The department should be prepared to face the large problems of policy which I have outlined in a letter to your Chairman which, I trust, will be read at your meeting tonight. Obviously those problems will not be solved intelligently and equitably if they are not approached with a vision directed to the loyalties of one’s subject and university rather than to self. Is it asking too much to relegate the personal aspects of this issue to oblivion?

It seems to me very important so to do. I have taken a great deal of pride in the distinction of the Department of Economics at Harvard and I have spoken in many circles boastfully of having what seems to me one of the very few remaining great departments of economics in the world. Certainly the responsibility of keeping that department great and of enabling it to develop continuing leadership should be the major loyalty to which every other consideration is subordinate. The awareness of this responsibility and the opportunities it presents will preoccupy your time and energies. Let me conclude by saying that I have always had and retain confidence in the intelligence, initiative, devotion. and cooperative spirit of your membership. I write this with all the more assurance because I know so many of you intimately and appreciate from personal friendship the qualities I have mentioned.

Very truly yours,
[signed]
Paul H. Buck

 

Source: Harvard University Archives. Department of Economics, Correspondence & Papers 1902-1950. Box 25. Folder “Graduate Instruction in Theory. Economics 101. 1942-43.

Categories
Exam Questions Johns Hopkins Undergraduate

Johns Hopkins. Undergraduate economics examinations, 1921-1922

 

Mid-year and year-end exams for the undergraduate political economy courses at Johns Hopkins for the academic year 1921-1922 have been transcribed for this post. Exams for the second semester of Political Economy V and VI were not found in the department’s file of old examinations. Names of instructors with their educational backgrounds along with short course descriptions are provided below as well.

_______________________________

Previous years’ exams transcribed

Undergraduate exams for 1919-20.

Undergraduate exams for 1922-23.

Undergraduate exams for 1923-24.

_______________________________

Johns Hopkins Faculty 1921-22
For Undergraduate Courses in Political Economy

Weyforth, William Oswald, Ph.D., Associate in Political Economy.

A.B., Johns Hopkins University, 1912, and Ph.D., 1915; Instructor, Western Reserve University, 1915-17.

Mitchell, Broadus, Ph.D., Instructor in Political Economy.

A.B., University of South Carolina, 1913; Fellow, Johns Hopkins University, 1916-17, and Ph.D., 1918.

Barnett, George Ernest, Ph.D., Professor of Statistics.

A.B., Randolph-Macon College, 1891; Fellow, John Hopkins University, 1899-1900, and Ph.D., 1901.

Jacobs, Theo, A.B., Associate in Social Economics.

A.B., Goucher College, 1901; Federated Charities of Baltimore (District Assistant, 1905-07, District Secretary, 1907-10, Assistant General Secretary, 1910-17, Acting General Secretary, 1917-19).

Sources:

Academic Rank  in 1921-22 from The Johns Hopkins University Circular, New Series 1922, No. 7. Report of the President of the University 1920-1921  (November 1922), p. 70.

Academic biographical data from The Johns Hopkins University Circular, University Register 1922-1923, No. 342, January 1923. Announcements for 1923-1924.

_______________________________

UNDERGRADUATE COURSES
1921-22

Political Economy I. Three hours weekly, through the year. In the first half-year the economic development of England and the industrial experience of the United States were studied. In the second half-year particular attention was given to the history of distribution and its application to leading economic problems. (Dr. Weyforth and Dr. Mitchell.)

Political Economy II. Three hours weekly, through the year. In the first half-year a preliminary study of the value and place of statistics as an instrument of investigation was made; attention was directed to the chief methods used in statistical inquiry. In the second half-year the principles of monetary· science were taught with reference to practical conditions in modern systems of currency, banking and credit. (Professor Barnett and Dr. Weyforth.)

Political Economy IV. Three hours weekly, through the year. In the first half-year the problems growing out of modern industrial employment were studied. In the second half-year the history of the industrial corporation was studied. (Professor Barnett and Dr. Mitchell.)

Political Economy VI. Three hours weekly, through the year. In the first half-year the applications of statistics to business and economic problems, such as price levels, cost of living, wage adjustments, business cycles, and business forecasting, were considered. In the second half-year the theory and practice of finance was considered, with particular reference to the problems of taxation presented in the experience of the United States. (Dr. Weyforth and Dr. Mitchell.)

Political Economy VII. Two hours weekly, through the year. History and development of social work. The responsibility of the State and private organizations toward the dependent, defective, and delinquent. (Miss Theo Jacobs.)

SourceThe Johns Hopkins University Circular, New Series 1922, No. 7. Report of the President of the University 1921-1922  (November 1922), pp. 56-57.

_______________________________

THE JOHNS HOPKINS UNIVERSITY
POLITICAL ECONOMY I
(Academic Section)

January 30, 1922 — 2-5 P.M.

  1. What was the economic situation of England during the Roman occupation?
  2. Describe the economic strength and weakness of the manorial system, and show how the feudal plan suggests the Single Tax scheme.
  3. What elements in gild life would be welcome in our present industrial order, and what elements of the medieval arrangement would be impossible with us at present?
  4. Tell what you know of trading in England in the middle ages.
  5. Suppose half the people of the United States should die inside of two or three years. What would be the chief economic consequences?
  6. Trace the gradually developing economic freedom of the lowest order of workers in England. Did peasants benefit more from the breaking up of the manorial system, or journeymen and apprentices from the collapse of the gilds?
  7. What was the economic condition of England on the eve of the Industrial Revolution?
  8. What is the significance of the Industrial Revolution? How did the factor system differ from the factory system?
  9. Define briefly: enclosures, Peasants’ revolt, Gresham’s Law, Steelyard, steward, serf or villein, apprentice, domestic system, Doomsday Book, Statute of Artificers, staple, virgate.
  10. What is the chief thing you have learned in this semester?

 

JOHNS HOPKINS UNIVERSITY
EXAMINATION IN POLITICAL ECONOMY I
(Academic Section)

Wednesday, May 31 — 2-5 P.M.

  1. What distinction would you draw between history, political science, and political economy? Explain fully.
  2. What facts in the industrial history of England illustrate economic principles that we have dwelt upon?
  3. Define: Wealth, capital, labor, time discount, wages of superintendence, consumer’s surplus, real wages, economic good, marginal productivity, entrepreneur.
  4. Explain carefully the differential principle of rent. With whose name do we link this theory, and how did Henry George employ the law of rent to justify the Single Tax?
  5. What was the wage-fund theory, and how was it used to discourage trade unionism?
  6. Comment fully on this passage from Adam Smith: “Nothing is more useful than water; but it will purchase scarce anything; scarce anything can be had in exchange for it. A diamond, on the contrary, has scarce any value in use, but a very great quantity of goods may frequently be had in exchange for it.”
  7. What accounts for the phenomenon of interest?
  8. What is meant by pure profit?
  9. Comment upon the decision of the Supreme Court of the United States in declaring the national act imposing a 10% tax on the net profits of industries employing children unconstitutional.
  10. Give, briefly, as many arguments as you can for and against trade unionism?
  11. State the number of firms interviewed by you in connection with the survey of the industrial life of the Negro in Baltimore.

RE-EXAMINATION
POLITICAL ECONOMY I.
[Handwritten note: Late June 1922]

  1. Define the following terms: “entrepreneur”, “marginal utility”, “capital”, “labor”, “diminishing returns”.
  2. Explain fully the differential principle of rent.
  3. Name and describe briefly four theories of wages.
  4. What in your judgment is the best justification for trade unions?
  5. What seems to you the most reasonable theory of interest?
  6. Explain the theory of value to which most emphasis was given in the lecture.

_______________________________

POLITICAL ECONOMY 1.
Dr. Weyforth

Monday — January 30, 1922 — Afternoon.

  1. Describe the chief characteristics of the economic life of the towns in England during the sixteenth and seventeenth centuries.
  2. What was the industrial revolution? What new conditions and what problems in economic life resulted from it?
  3. Define the following terms: goods, free goods, economic exchange value, price,
  4. Explain the underlying principles of “scientific management” in production. State and explain the attitude of organized labor toward scientific management.
  5. Explain how market price is determined under conditions of competition. What is the relationship between market price and expenses of production?
  6. Describe the principal forms of combination that have been used in the United States. Outline the main features of Federal legislation concerning combinations.
  7. What is meant by standard money? What are the requirements of a bimetallic standard? Outline the main features of the monetary legislation of the United States.
  8. What is a corporation? How is it brought into existence? What are its advantages as compared with the partnership or individual enterprise? Describe the principal securities through the issue of which its capital is obtained.

 

JOHNS HOPKINS UNIVERSITY
POLITICAL ECONOMY 1
(Engineering Group)

[N.B. falsely filed as a 1923 exam]

Wednesday, May 31.

  1. Define the various types of credit. Explain how bank credit serves as a substitute for money as a medium of exchange,
  2. Explain the factors that a bank officer takes into consideration in judging of the credit standing of a borrower,
  3. What is the fallacy involved in the mercantilist theory of the desirability of a favorable balance of trade?
  4. Explain the theory that each factor in production tends to receive a share of the product corresponding to its marginal productivity.
  5. What is interest? Give an analysis of the forces that determine its rate.
  6. How do you account for inequalities in the personal distribution of wealth? Why is less inequality desirable? How could it be effected?
  7. What are some of the outstanding economic characteristics of railroad transportation? Explain their bearing upon the following: (a) practice of charging what the traffic will bear; (b) large variations in net earnings with small variations in traffic; (c) cut-throat nature of competition that sometimes develops.
  8. What is socialism? Give briefly the arguments for and against

_______________________________

POLITICAL ECONOMY II.

Thursday, February 2, 1922 — 9-12 A.M.

  1. What is the distinction between a census and a registration?
  2. Define an average. Illustrate by defining the arithmetic mean, the mode and the median.
  3. Define an index number. Explain the difference between the aggregate and the relative methods of constructing an index number.
  4. Taking the following group of figures calculate the standard deviation:
Height of men No. in Class
5.6 — 5.7 28
5.7 — 5.8 42
5.8 — 5.9 65
5.9 — 5.10 78
5.10 — 5.11 164
5.11 — 6.0 92
6.0 — 6.1 46
6.1 — 6.2 7
  1. For the same group, calculate the mode.
  2. For the same group, calculate the mean.

JOHNS HOPKINS UNIVERSITY
POLITICAL ECONOMY II
Money and Banking
Dr. Weyforth

Friday, June 2

  1. What is bimetallism? What are the chief requisites of a bimetallic standard? What principles do the bimetallists depend for maintaining the concurrent circulation of gold and silver?
  2. Define credit. What are the various kinds of credit? Distinguish especially the difference between investment credit and mercantile or commercial credit?
  3. What is the function of the commercial paper house or note broker in present day commercial banking?
  4. What is (a) a trade acceptance and (b) a bank acceptance? Explain their use and advantages.
  5. What problems are presented to bankers (a) by seasonal fluctuations in business and (b) by cyclical fluctuations in business?
  6. Describe the organization of the Federal Reserve System.
  7. In what way does the Federal Reserve System provide for elasticity in currency and elasticity in credit?
  8. What is the principle that governs the distribution of gold among the nations of the world under normal conditions such as those existing before the war?

_______________________________

THE JOHNS HOPKINS UNIVERSITY
POLITICAL ECONOMY IV
(Labor Problems)

February 2, 1922 — 9 A.M.-12M.

  1. What are the principal reasons for believing that trade unionism and employers’ associations did not originate in the medieval gilds?
  2. State in some detail who Francis Place was and explain his service to trade unionism.
  3. What present-day evidences have we of the spirit which characterized the English combination acts?
  4. State the arguments for and against the “closed shop”.
  5. What do you know of the history of strikes?
  6. Give your estimate of the purposes and progress of workers’ education in England and in the United States?
  7. In the light of what you have learned, do you believe compulsory arbitration likely to promote industrial peace? What would you make the main provisions of a compulsory arbitration law could such be passed by congress?
  8. Speak of the trade agreement and its significance.
  9. Describe briefly one of the books you read during this course.
  10. What do you think will be the next important development in the labor movement in this country?
  11. List the books you have read for this course.

 

EXAMINATION IN POLITICAL ECONOMY IV.

Friday, June 2nd, 1922, (3-5 p.m.)

  1. What are the advantages and disadvantages of the partnership?
  2. Why is a complex capitalization usually superior to a simple form of capitalization?
  3. What is the distinction between preferred and common stock as to (a) income, (b) control, (c) risk?
  4. Define mortgage bonds, debenture bonds, income bonds, collateral bonds and equipment trust bonds.
  5. What is meant by amortization? Under what circumstances is some provision for amortization necessary for the protection of the bond-holders?
  6. What is the distinction between an underwriting syndicate with undivided liability and a syndicate with divided liability?
  7. Does a stock dividend theoretically increase the total value of the stock outstanding? Practically how does it frequently work and why?
  8. What are the advantages of the holding company form of organization?

 

_______________________________

POLITICAL ECONOMY VI
Dr. Weyforth.

Tuesday — January 31, 1922 — Afternoon.

  1. Explain the construction of a logarithmic chart. What are its advantages?
  2. Explain and illustrate the construction of (a) an index number of relatives, and (b) an index number of aggregates. What advantages are claimed for the latter?
  3. Describe the way in which the Bureau of Labor Statistics index number of the total cost of living is constructed
  4. What is the utility of an index number of the physical volume of production? Explain how Professor Stewart and Professor Day respectively constructed their index numbers.
  5. Explain as fully as you can the system employed by the Harvard University Committee on Economic Research for the forecasting of business conditions.

_______________________________

[POLITICAL ECONOMY VII.]
SOCIAL ECONOMICS
Miss Jacobs.

Monday — January 30, 1922  — Morning

  1. Give the arguments for and against public outdoor relief.
  2. Give the war and peace time activities of the American Red Cross.
  3. What is the Confidential Exchange of Information? What is its value to the community?
  4. What are the effects of dependency and delinquency upon the community?
  5. Give some of the causes of poverty. Tell how some of them may be lessened or eradicated.
  6. Give the objects and aims of three (3) social organizations that seem the most important to you.

 

Source: Johns Hopkins University. Eisenhower Library. The Ferdinand Hamburger, Jr. Archives. Department of Political Economy, Series 5/6. Box No. 6/1, Folder “Exams 1907-1924.”

Image Source. Gilman Hall image from the 1924 edition of the Johns Hopkins’ yearbook Hullabaloo.

 

Categories
Harvard Suggested Reading Syllabus

Harvard. Junior Honors Reading Lists. Conrad and Henderson, 1958-1959

 

Before the 1958-59 academic year began, tragedy struck the Harvard economics department. Assistant professor of economics Stefan Valavanis (31 years old) was found shot in his tent near Mt. Olympus. He was to be the lead instructor for the Junior year honors course in economics during the up-coming year. With his loss the course was left to his assistant professor colleagues, Alfred Haskell Conrad and James Mitchell Henderson.

Alfred Haskell Conrad, Ph.D. 1954. The Redistribution of Incomes and the Matrix Multiplier: The Impact of Fiscal Policy on the Distribution of Income in 1950. Advisors: W.W. Leontief and John S. Chipman. (Mathematics Genealogy Project)

James Mitchell Henderson, Ph.D. 1955. The Efficiency of the Coal Industry: An Application of Linear Programming. Advisors: W.W. Leontief and Elizabeth Waterman Gilboy. (Mathematics Genealogy Project)

James M. Henderson was co-author with Richard Quandt of Microeconomic Theory: A Mathematical Approach, the leading graduate microeconomics text in its day and (fun fact) married Anne O. Krueger in July 1981 when they both were professors at the University of Minnesota. The two of them moved on to become professors at Duke University. Henderson died in 1992.

_____________________

Course Announcement

Economics 100. Junior Honors Course

Full course. M., 4-6. Assistant Professors [Stefan] Valavanis (in charge), [Alfred Haskell] Conrad and (spring term) [James Mitchell] Henderson.

Permission required of Assistant Professor Valavanis.

Required course for Economics concentrators who are candidates for honors. This course will deal with the theory of wages and prices; problems of public policy in the fields of Industrial Organization and Labor; the relation between descriptive material and theoretical analysis; methods of testing hypotheses. One two-hour group meeting each week organized as a seminar with papers by students. Additional papers and individual conferences with staff members.

Source: Courses of Instruction Offered by the Faculty of Arts and Sciences 1958-1959. Official Register of Harvard University, Vol. LV, No. 20 (September 3, 1958), p. 89.

_____________________

Course Enrollment

[Economics] 100. Junior Honors Course. Assistant Professors Conrad and (S) Henderson. Full course.

(F) Total 49: 3 Seniors, 40 Juniors, 3 Sophomores, 3 Radcliffe
(S) Total 46: 4 Seniors, 37 Juniors, 2 Sophomores, 3 Radcliffe

Source: Harvard University. Report of the President of Harvard College 1958-59, p. 70.

_____________________

Final Reading List and Course Outline
Economies 100
Fall, 1958

I. Introduction: Some background and material on the nature and setting of American industrial enterprise. The market mechanism.

W. Adams (ed.), The Structure of American Industry, Chs. 5, 6, 7, 11, 13.

A.B. Jack, “The Channels of Distribution for an Innovation,” Explorations in Entrepreneurial History, IX, p. 113.

K.E. Boulding, Economic Analysis, Part I (any edition).

II. The Empirical Measurement of Economic Functions.

M.J. Moroney, Facts from Figures, Ch. 2, 4; (probability and central tendency).

R.E. Freund, Modern Elementary Statistics, Ch. 12, 13.

Jan Tinbergen, Econometrics, Chs. 1, 2, 5.

R. G. D. Allen, Mathematical Analysis for Economists, Ch. 2; Ch. 6 (recommended) (functions and derivatives).

III. Factors Determining Industry Structure.

A. Production Theory

E. A. G. Robinson, The Structure of Competitive Industry, Chs. 1-5.

T. Scitovsky, Welfare and Competition, Chs. 6,7.

V. E. Smith, “The Statistical Production Function,” Quarterly Journal of Economics, 1944-45, pp. 543-62.

R. G. Bressler, Jr. “Research Determination of Economies of Scale,” Journal of Farm Economics, 1945, p. 526.

B. Cost Curves

Committee on Price Determination for the Conference on Price Research, Cost Behavior and Price Policy, pp. 80-115, 291-301, 219-263, 321-329.

J. Viner, “Cost Curves and Supply Curves,” Readings in Price Theory. Ch. 10, p. 198.

M. Colberg, et al, Business Economics, Ch. 4, 5.

C. Barriers to entry other than economies of scale

J. Bain, Barriers to New Competition, Ch. 6.

IV. Markets and Pricing:

A. Indifference Curves and Consumer Theory

George J. Stigler, The Theory of Price (any edition) Ch. 5.

Scitovsky, Welfare and Competition, Chs. 2, 3, 4.

Boulding, Economic Analysis, Ch. 34 (revised edition).

B. Pure Competition and Pure Monopoly

E. H. Chamberlin, Theory of Monopolistic Competition, Chs. 1, 2.

T. Scitovsky, Welfare and Competition, Chs. 8, 16.

C. Oligopoly

Chamberlin, Monopolistic Competition, Chs. 3, 4, 5.

Readings in Price Theory, Chs. 20, 21.

T. Scitovsky, Welfare and Competition, Ch. 20.

R. M. Alt, “Statistical Measurement of Price Flexibility,” Quarterly Journal of Economics, 1949, p. 92.

J. Bain, Barriers, Ch. 7.

J. Vanek, “The Nature of Equilibrium in Monopolistic Competition,” Mimeographed.

V. Investment Theory

A. An outline of formal theory

Samuelson, Economics, Ch. 29.

Boulding, Chs. 35-37 (revised edition).

Joel Dean, Capital Budgeting, Chs. 1-3.

J. M. Clark, “Business Acceleration and the Law of Demand,” Readings in Business Cycle Theory.

J. Meyer and E. Kuh, The Investment Decision, Ch. 11 (2).

B. Uncertainty

R. Weckstein, “On the Use of the Theory of Probability in Economics,” Review of Economics and Statistics, 1953, pp. 191-198.

Ward Edwards, “The Theory of Decision Making,” Psychological Bulletin, 1954.

A. G. Hart, Anticipation, Uncertainty, and Dynamic Planning.

C. Financial Considerations

J. Meyer and E. Kuh, The Investment Decision, Chs. 9, 12.

R. Mack, The Flow of Business Funds and Consumer Purchasing Power, Ch. VIII.

W. Heller, “The Anatomy of Investment Decisions,” Harvard Business Review, March 1951.

Fortune, “The Fine Art of Raising Capital,” u, July 1956 and ”How Much Can Business Borrow?” June 1956.

D. The Influence of Market Structure on the Investment Decision and Innovation.

W. Fellner,” The Influence of Market Structure on Technological Progress,” Quarterly Journal of Economics, November 1951, pp. 556-577.

C. Kaysen, “A Dynamic Aspect of the Monopoly Problem,” Review of Economics and Statistics, May 1949, pp. 109-113.

Schumpeter, Capitalism, Socialism and Democracy, Ch. VIIl, pp. 87-106.

_____________________

Economics 100
Reading List
Spring, 1959

I. General Equilibrium.

E. Phelps Brown, The Framework of the Pricing System, Chaps. II-IV.

F. Zeuthen, Economic Theory and Method, Chaps. 10-12 and pp. 194-197.

J. R. Hicks, Value and Capital, Part II.

W. W. Leontief, “Input-Output Economics,” Scientific American, October, 1951.

W. W. Leontief, “Input-Output Analysis and the General Equilibrium Theory,” in T. Barna, ed., The Structural Interdependence of the Economy, pp. 42-49.

Selected Applications

(The material listed under this heading may be used as the basis for tutorial papers and discussion. Specific assignments will be made by the instructor.)

W. W. Leontief, “Factor Proportions and the Structure of American Trade,” Review of Economics and Statistics, Nov., 1956.

R. S. Eckaus, “The Factor Proportions Problem in Underdeveloped Areas,” American Economic Review, September, 1955.

II. Welfare Economics.

T. Scitovsky, Welfare and Competition, Chaps. 4, 8, 16. (for review).

J. de V. Graaff, Theoretical Welfare Economics. Chaps. I-V, X, XI. (omit the appendices)

K. Boulding, “Welfare Economics,” in Survey of Contemporary Economics, Vol. II, B. F. Haley, ed.

A. P. Lerner, The Economics of Control, Chaps. 3, 4.

Selected Applications

M. Friedman, in R. Solo, ed., Economics and the Public Interest, Chap. 9.

W. S. Vickey, The Revision of the Rapid Transit Fare Structure of the City of New York, (hectographed; in Littauer Library).

J. V. Krutilla and O. Eckstein, Multi-Purpose River Development, 1958.

N. Kaldor, The Expenditure Tax.

III. Macro ModelsCycles, Growth, Money.

G. Haberler, Prosperity and Depression, Chaps. 2, 3, 5.

A. Hansen, Business Cycles and National Income, Part Il.

A. Hansen, Monetary Theory and Fiscal Policy, Chaps. 3-6.

J. S. Duesenberry, Business Cycles and Economic Growth, Chaps. 2-5, 9-12.

C. Christ, “Aggregate Econometric Models,” American Economic Review, June, 1956

H. Makower and J. Marschak, “Assets, Prices and Monetary Theory,” in G. Stigler and K. Boulding, eds., Readings in Price Theory. Chap. 14.

J. Gurley and E. Shaw, “Financial Aspects of Economic Development,” American Economic Review, September, 1955.

Selected applications from the fields of wage and price policy, inflation, international trade and development will be assigned by the instructors.

Source: Harvard University Archives. Harvard University: Syllabi, course outlines and reading lists in Economics, 1895-2003. Box 7, Folder “Economics, 1958-59 (1 of 2).”

Image Sources:

Left. 1959 Alfred Haksell Conrad, John Simon Guggeheim Memorial Foundation.
Right:  1959, James M. Henderson, University of Minnesota Archives.

Categories
Exam Questions Harvard

Harvard. Examinations for empirical economics, Guy Orcutt, 1950, 1951

 

The exams transcribed and posted below add to the historical record of Guy Orcutt‘s early attempts at Harvard to introduce undergraduates and graduates to econometrics.

________________________________

Previously posted

The bibliography of books and articles on the scientific method for Economics 110 by Guy Orcutt.

The second batch of material from Guy Henderson Orcutt’s undergraduate course Economics 110 at Harvard from 1959-50.

________________________________

Empirical Economics (spring term) 1949-50

Economics 110. Announcement, 1949-50

Economics 110. Introduction to Econometrics

Half-course (spring term). Mon., Wed., Fri., at 9. Dr. ——-

The matter will be presented in order of increasing mathematical difficulty. Only simplified models will be used to familiarize the students with the econometric approach: and to complete their knowledge of mathematical tools needed in quantitative economic analysis.

Source. Final Announcement of the Courses of Instruction Offered by the Faculty of Arts and Sciences During 1949-50. Official Register of Harvard University, Vo. XLVI, No. 24 (September, 1949) , p. 79.

*  *  *  *  *  *  *  *  *  *  *

Course Enrollment

Economics 110. Applied Economics. Enrollment 1950

[Economics] 110 Applied Economics. (Sp) Assistant Professor Orcutt.

Total: 6:  1 Graduate, 2 Seniors, 2 Juniors, 1 Sophomore.

Source: Harvard University, Report of the President of Harvard College and Reports of Departments for 1949-50, p. 72.

*  *  *  *  *  *  *  *  *  *  *

1949-50
HARVARD UNIVERSITY
APPLIED ECONOMICS – ECONOMICS 110
[Final Examinations]

(Three Hours)

Do all of the following five questions:

  1. Describe what you consider to be the essential characteristics of the experimental method. In what respects do you consider the problem of making inferences from non-experimental data different from that of drawing inferences from experimental data?
  2. Describe one or two of the empirical studies dealing with the demand for imports or exports. Indicate in detail why the price elasticities arrived at in the empirical demand for import studies are subject to considerable error and may very well be substantial underestimates.
  3. What do you consider to be the role of theory in economics? Be as specific as possible.
  4. What are null hypotheses and what role do they play in testing hypotheses? Describe the nature of the null hypothesis generally used in testing the significance of correlations between series of data. Why is the usual test inappropriate when dealing with correlations between economic time series? What are some of the more appropriate methods for testing the significance of correlations between economic time series?
  5. Comment on either An Introduction to Econometrics by Jacob Marschak, or on one of the books dealing with methodology in economics.

Source:  Harvard University Archives. Syllabi, course outlines and reading lists in Economics, 1895-2003. Box 27. Papers Printed for Final Examinations [in] History, History of Religions, … , Economics, … , Military Science, Naval Science, June 1950.

________________________________

Empirical Economics (fall term) 1950-51

Course Announcement

Economics 110a. Empirical Economics: National Income and Business Fluctuations
Half-course (fall term). Tu., Th., and (at the pleasure of the instructor) Sat., at 12. Assistant Professor Orcutt.

This course will deal with the empirical foundations of economic theory in the fields of national income and business fluctuations. The methods by which various types of prediction are attempted will be given considerable attention.

[Economics 110b. Empirical Economics: The Price Mechanism]
Half-course (fall term). Tu., Th., and (at the pleasure of the instructor) Sat., at 12. Assistant Professor Orcutt.
Omitted in 1950-51; to be given in 1951-52.   [NOTE. ALSO OMITTED IN 1951-52]

This course will deal with the empirical foundations of economic theory concerning the functioning of the price mechanism. The agricultural and foreign trade sections will receive particular attention. Properly qualified undergraduates will be admitted to Economics 210b.

Source: Harvard University, Courses of Instruction Offered by the Faculty of Arts and Sciences, 1950-51. Official Register of Harvard University, Vol. XLVII, No. 23 (September, 1950), p. 80.

*  *  *  *  *  *  *  *  *  *  *

1950-51
HARVARD UNIVERSITY ECONOMICS 110a
[Final Examination, Mid-Year]

  1. Write for about 30 to 45 minutes on business cycle theory. In your discussion be sure to include a thorough treatment of both the acceleration principle (or principle of derived demand) and the multiplier analysis. Be as explicit as possible about their ranges of possible or suggested applications and about their limitations.
  2. Write for about 30 to 45 minutes on the problem of testing the significance of correlation coefficients. In the course of your discussion bring out the meaning of the correlation coefficient, the nature of tests of significance, the basis of the standard test of significance, reasons why the standard test is frequently not appropriate for testing the significance of correlations between economic time series, and some of the ways of attempting to deal with the problem of testing the significance of correlations between economic time series.
  3. Write for about 45 to 60 minutes on the problems involved in the statistical determination of the consumption function. Include in your discussion the difficulties arising from additional relationships, errors of observations, and auto-correlated error terms. Also discuss some of the difficulties involved in using cross-section data in testing for the effect of assets on consumption.
  4. Spend about 30 minutes in making out a list of problems that you think should be worked on in business cycles research.
  5. Indicate what you think is meant by the following terms or phrases:

(a) structural relations
(b) Exogenous variables
(c) endogenous variables
(d) X is a cause of Y

Source: Harvard University Archives. Harvard University Final Examinations, 1853-2001. Box 17, Bound volume: Final Exams—Social Sciences, Jan. 1951. Papers Printed for Final Examinations [in] History, History of Religions, … , Economics, … ,Military Science, Naval Science. January, 1951.

Image Source: Orcutt’s senior year picture from the University of Michigan yearbook, Michiganensian, 1939.

Categories
Exam Questions Harvard

Harvard. Economics and Social Ethics Semester Examinations, 1895-96

 

Professor Charles Dunbar had a leave of absence for the 1895-96 academic year at Harvard. His courses on public finance were taught by Dr. John Cummings and Dr. J.A. Hill. Professor Frank Taussig returned from his year leave of absence for 1894-95 and taught (among other courses) the history of financial institutions, Dunbar’s second field of specialization.

___________________________

1895-96.
The Ethics of the Social Questions.

Course Enrollment for Philosophy 5

For Graduates and Undergraduates:—

[Philosophy] 5 . Professor Peabody. — The Ethics of the Social Questions. — The problems of Poor-Relief, the Family, Temperance, and various phases of the Labor Question in the light of ethical theory. — Lectures, special researches, and required reading. 3 hours.

Total 88: 7 Graduates, 49 Seniors, 12 Juniors, 2 Sophomores, 18 Others.

Source: Harvard University. Report of the President of Harvard College, 1895-96, p. 60.

*  *  *  *  *  *  *  *  *  *

Mid-Year Examination

1895-96.
PHILOSOPHY 5.
THE ETHICS OF THE SOCIAL QUESTIONS.

[This paper should be considered as a whole. The time should not be exhausted in answering a few questions, but such limit should be given to each answer as will permit the answering of all the questions in the time assigned.]

  1. What is meant by;

Exogamy;
Marriage by capture;
The Patriarchal theory;
“The family is the unit of civilization”?

  1. The stability of the family as affected by:

(a) city life.
(b) the conflict of State laws.
(c) the philosophy of individualism.
(d) the philosophy of collectivism.

  1. Spencer’s view of the regime of the family in relation to the regime of the State (Principles of Sociology I, 707 pp.), with criticisms.
  2. The distribution of wealth in Great Britain or in the United States, statistically illustrated; and its lessons,
  3. Illustrate the indirect economic value of judicious charity.
  4. Charles Booth’s Class B in East London; its character, dimensions, relation to the general problem of poverty, and suggested treatment. Life and Labor of the People, I, 39-44; 162-169.)
  5. The new inquiry undertaken by Mr. Charles Booth (Vol. V and VI, 1895); its relation to the preceding researches and its confirmation of earlier results.
  6. “What is good in the poor-administration of Germany is due to good citizenship. … We have not citizenship enough to administer it.” (C. S. Loch, Parliamentary Report of 1888, p. 88.) Compare, in the light of this comment, the English and German theories of municipal relief.
  7. The influx to the great cities in its effect on methods of poor-relief.

Source: Harvard University Archives. Harvard University, Mid-year Examinations 1852-1943. Box 3, Bound volume Examination Papers, Mid-Years 1895-96.

*  *  *  *  *  *  *  *  *  *

Final Examination

1895-96.
PHILOSOPHY 5.
THE ETHICS OF THE SOCIAL QUESTIONS.

[This paper should be considered as a whole. The time should not be exhausted in answering a few questions, but such limit should be given to each answer as will permit the answering of all the questions in the time assigned.]

  1. Explain and illustrate, briefly, what is meant by:

“The social questions are ethical questions.”
“The correlation of the social questions.”

  1. The doctrines of social progress in Carlyle and in Ruskin compared in their bearing on the modern industrial situation.
  2. Consider the principle of social labour-time as the standard of value:

(a) Mr. Ruskin’s theory of value;
(b) The plan proposed by scientific socialism (Schäffle, p. 81);
(c) Schäffle’s criticism of this view (ch. VI., VII.);
(d) Your own judgment.

  1. “Socialism has no necessary affinity with any forms of violence, or confiscation, or class selfishness, or financial arrangement. … The aim of socialism is the fulfilment of service; the aim of individualism is the attainment of some personal advantage, riches, or place, or fame.” — Bishop Westcott.
    “Socialism, as I understand it, is any theory of social organization which sacrifices the legitimate liberties of individuals to the will or interest of the community.” — Professor Flint.
    Which of these definitions appears to you more justified by the history and tendency of socialism? What do you understand to be the “quintessence” of socialism?
  2. The economic and ethical criticisms commonly urged against the programme of collectivism, and your estimate of their importance.
  3. The ethical place and lessons of:

Anarchism;
Communism;
Arbitration.

  1. Compare the plans of industrial unity illustrated by the Anzin collieries, the Val-des-Bois Mill, and the Hebden Bridge Mill.
  2. The coöperative movement in Great Britain, its principles, its expansion, and the conditions of success for the system in this country. In federalistic coöperation what should be, in your judgment, the principle of distributing the bonus?
  3. The polities and the ethics of the Maine liquor law (Fanshawe, VII.)

Source: Harvard University Archives. Harvard University, Examination papers 1873-1915. Box 4, Bound volume: Examination Papers 1896-97. Papers Set for Final Examinations in Philosophy, History, Government, Economics, Fine Arts, Architecture, and Music in Harvard College, June 1896.

___________________________

1895-96.
Outlines of Economics.

Course Enrollment for Economics 1

Primarily for Undergraduates:—

[Economics] 1. Professors Taussig and Ashley, Asst. Professor Edward Cummings, and Dr. John Cummings. — Outlines of Economics. — Mill’s Principles of Political Economy. — Lectures on Economic Development, Distribution, Social Questions, and Financial Legislation. 3 hours.

Total 338: 3 Graduates, 35 Seniors, 91 Juniors, 161 Sophomores, 8 Freshmen, 40 Others.

Source: Harvard University. Report of the President of Harvard College, 1895-96, p. 63.

*  *  *  *  *  *  *  *  *  *

Mid-Year Examination

1895-96.
ECONOMICS 1.

  1. Is all wealth produced by labor?
  2. Compare the distinction between fixed and circulating capital with the distinction between auxiliary and remuneratory capital; and state why one or the other distinction is the more satisfactory.
  3. Are differences in profits from employment to employment similar in kind to differences in wages from occupation to occupation?
  4. In what way are differences of wages affected by the absence of effective competition between laborers? by its presence?
  5. What are the grounds for saying that rent is a return differing in kind from interest?
  6. Trace the effects of an issue of inconvertible paper money, less in quantity than the specie previously in use, on (1) the circulation of specie, (2) the foreign exchanges, (3) the relations of debtor to creditor.
  7. State Mill’s reasoning as to the mode in which, under a double standard, one metal is driven from circulation; and explain how the actual process differs from that analyzed by Mill.
  8. What are the grounds for saying that the gain of international trade does not come from the sale of surplus produce beyond the domestic demand?
  9. In what manner is the price of landed property affected by an increased quantity of money? by a rise in the rate of interest?
  10. Wherein does monopoly value present a case different from that of the usual operation of the laws of value?

Source: Harvard University Archives. Harvard University, Mid-year Examinations 1852-1943. Box 3, Bound volume Examination Papers, Mid-Years 1895-96.

*  *  *  *  *  *  *  *  *  *

Final Examination

1895-96.
ECONOMICS 1.

[Answer ten questions. Arrange your answers strictly in the order of the questions.]

GROUP I.
[At least one.]

  1. Explain the meaning of two of the following terms, — margin of cultivation; wages of superintendence; rapidity of circulation (as to money).
  2. Do profits constitute a return different from interest?
  3. Explain what is meant by the law, or equation, of demand and supply; and in what manner it applies to commodities susceptible of indefinite multiplication without increase of cost.
  4. In what manner does a country gain from the division of labor in its domestic trade? In what manner from international trade?

GROUP II.
[At least one.]

  1. Does it fall within the province of the economist to discuss the institution of private property?
  2. Show the connection between the industrial development of the present century, and the discussion among economists as to the functions of the entrepreneur.
  3. Consider in what manner prices, or rents, [choose one] are differently determined according as they are under the influence of custom or of competition.
  4. “The idea that economic life has ever been a progress mainly dependent on individual action is mistaken with regard to all stages of civilization, and in some respects it is the more mistaken the farther we go back.” Explain and criticize.

GROUP III.
[At least one.]

  1. If coöperation were universally adopted, what would be left of the wages system?
  2. Is there anything in what you learned as to the laws governing wages, which the action of the English trade-unions in regard to wages has disregarded?
  3. Has the course of events justified Mill’s expectations in regard to the development of profit-sharing and of cooperation? Explain why, or why not.
  4. Describe the trade and benefit features of the English trade-unions.

GROUP IV.
[At least three.]

  1. Is the present position of the Treasury of the United States in any respect essentially similar to that of the Issue Department of the Bank of England? In any respect essentially dissimilar?
  2. What is the test of over-issue, as to inconvertible paper money? What light does the experience of the United States and of France throw on the probability of over-issue?
  3. Arrange in their proper order the following items in a bank account:
Capital 100,000 Bonds and Stocks 75,000
Specie 150,000 Surplus 50,000
Notes 100,000 Other Assets 50,000
Loans 400,000 Other Liabilities 60,000
Expenses 25,000 Undivided Profits 40,000
Deposits 350,000

Could this bank be a national bank of the United States? If such a bank, how would the account stand?

  1. Compare the policy of the Bank of England in times of financial crisis with the policy of the Associated Banks of New York; and give an opinion as to which is the more effective in allaying panic.

Source: Harvard University Archives. Harvard University, Examination papers 1873-1915. Box 4, Bound volume: Examination Papers 1896-97. Papers Set for Final Examinations in Philosophy, History, Government, Economics, Fine Arts, Architecture, and Music in Harvard College, June 1896.

___________________________

1895-96.
Economic Theory from Adam Smith to the present time.

Course Enrollment for Economics 2.

For Graduates and Undergraduates:—

[Economics] 2. Professors Ashley and Macvane. — Economic Theory from Adam Smith to the present time. — Selections from Adam Smith and Ricardo. — Modern Writers. —Lectures. 3 hours.

Total 37: 5 Graduates, 14 Seniors, 7 Juniors, 4 Sophomores, 7 Others.

Source: Harvard University. Report of the President of Harvard College, 1895-96, p. 63.

*  *  *  *  *  *  *  *  *  *

Mid-Year Examination.

1895-96.
ECONOMICS 2

N.B. — Not more than seven questions must be attempted.

  1. Compare the Aristotelian conception of Wealth with that of modern economists.
  2. Explain the growth, in the later Middle Ages, of the theory of “Interest.”
  3. Consider briefly the claims to consideration, in the history of economic thought, of Nicholas Oresme and Antoine de Montchrétien.
  4. “It was reserved for the eighteenth century to let in the grand idea of necessity, and to prove that the rate of wages established in a country was the inevitable consequence of the circumstances in which that country was placed, and had no connection with the wishes of any individual, or, indeed, with the wishes of any class.” (Buckle, History of Civilization.) Consider this.
  5. Explain the “plan” of the Wealth of Nations, and consider how far it agrees with the contents of the work.
  6. State and discuss Adam Smith’s doctrine of the Component Parts of Price.
  7. “A man must always live in his work.” Discuss the accuracy of this proposition, and the use made of it by Adam Smith and later economists.
  8. The effect upon English economists in the seventeenth and eighteenth centuries of their observation of the United Netherlands.
  9. “Every candid reader knows that Mr. Malthus laid no stress on his unlucky attempt to give numerical precision to things which do not admit of it, and every person capable of reasoning must see that it is wholly superfluous to his argument.” (Mill). Consider this.
  10. With what justice can socialists claim the authority of Ricardo for their “iron law of wages”?

Source: Harvard University Archives. Harvard University, Mid-year Examinations 1852-1943. Box 3, Bound volume Examination Papers, Mid-Years 1895-96.

*  *  *  *  *  *  *  *  *  *

Final Examination

1895-96.
ECONOMICS 2.

Take any three of the five questions.

  1. State your conclusions regarding the various definitions of Cost of Production. Are wages an element in Cost? Show whether economic cost and commercial (or employers’) cost may vary independently of each other.
  2. State briefly the views of Henry George, Marshall, and Boehm-Bawerk (or any other three writers) regarding the law of Interest. Give also your own conclusions.
  3. Set down carefully your conclusions as to the source and the law of Wages. Examine at least one opposing view.
  4. Explain and examine the Marginal Utility theory of Value. How is it reconciled with the observed connection between value and cost?
  5. Is a high level of wages in a country an obstacle to foreign trade?

Source: Harvard University Archives. Harvard University, Examination papers 1873-1915. Box 4, Bound volume: Examination Papers 1896-97. Papers Set for Final Examinations in Philosophy, History, Government, Economics, Fine Arts, Architecture, and Music in Harvard College, June 1896.

___________________________

1895-96.
The Principles of Sociology.

Course Enrollment for Economics 3.

For Graduates and Undergraduates:—

[Economics] 3. Asst. Professor Edward Cummings. — The Principles of Sociology. — Development of the Modern State, and of its Social Functions. 2 hours.

Total 37: 8 Graduates, 21 Seniors, 6 Juniors, 2 Others.

Source: Harvard University. Report of the President of Harvard College, 1895-96, p. 63.

*  *  *  *  *  *  *  *  *  *

Mid-Year Examination.

1895-96.
ECONOMICS 3.

(Arrange your answers in the order of your questions. Omit one.)

  1. “Hence, in this case we may assert clearly that when the individual is removed the social ceases to be, and that there is absolutely nothing in society which does not exist in a state of subdivision and continual repetition in the living individuals, — or which has not existed in the dead ancestors from whom the living proceed.” Explain carefully. Compare this conception of society with the “social organism” conception, and state clearly your own views.
  2. What do you conceive to have been the habits and characteristics of primitive man in “a state of nature”? Discuss the evidence presented by Westermarck, Spencer, and others.
  3. “In a word, the physiological bond, which of old constituted the main foundation of the small domestic societies, then of the tribes, then of the ancient cities, is still the essential foundation of the great nations of today.” Explain carefully. What according to Spencer, have been the merits and defects of the various forms of family organization? What are the present tendencies?
  4. “Entangled and confused with one another as Ceremonial and Fashion are, they have thus different origins and meanings.” Explain. Trace carefully the significance of these differences, and give examples.
  5. “Class distinctions, then, date back to the beginnings of social life.”
  6. In what order have political institutions evolved? What have been the chief determining factors?
  7. “M. Alfred Fouillée has endeavored to express the truth of both ways of regarding society by saying that the highest form of it must be an ‘organism contractuel,’ — a formula that may perhaps gain more general acceptance than anything expressed in the phraseology of German idealism.” Explain carefully.
  8. Discuss the views of Spencer and of Comte in regard to the scope of sociology and its relation to other sciences

Source: Harvard University Archives. Harvard University, Mid-year Examinations 1852-1943. Box 3, Bound volume Examination Papers, Mid-Years 1895-96.

*  *  *  *  *  *  *  *  *  *

Final Examination

1895-96.
ECONOMICS 3.

Answer the questions in the order in which they stand. Give one hour to each division.

I.

A critical estimate of Giddings’ Principles of Sociology, — contrasted (a) with Spencer, (b) with Tarde.

II.

A critical estimate of Evolution and Effort, — contrasting it with views set forth in Social Evolution.

III.

The bearing of sociological theory upon the practical problems of (a) poverty, (b) pauperism, (c) crime.
Which of the books read during this half-year (and not already discussed) has seemed to you of greatest worth? Why?

Source: Harvard University Archives. Harvard University, Examination papers 1873-1915. Box 4, Bound volume: Examination Papers 1896-97. Papers Set for Final Examinations in Philosophy, History, Government, Economics, Fine Arts, Architecture, and Music in Harvard College, June 1896.

___________________________

1895-96.
The Theory of Statistics.

Course Enrollment for Economics 42.

For Graduates and Undergraduates:—

[Economics] 42. Dr. John Cummings. — The Theory of Statistics. — Applications to Social and Economic Problems. — Studies in movements of Population. Hf. 3 hours. 2d half year.

Total 19: 2 Graduates, 11 Seniors, 4 Juniors, 2 Sophomores.

Source: Harvard University. Report of the President of Harvard College, 1895-96, p. 64.

*  *  *  *  *  *  *  *  *  *

Final Examination

1895-96.
ECONOMICS 4.

(Divide your time equally between A and B.)

A
I and II may be treated as one question.

  1. What do you understand by “movement of population”? What light do Statistics throw upon the law of population as stated by Malthus?
  2. What are some of the “more striking facts and more pregnant results of the vast growth of population in Europe, America, and the British Colonies within the last half century”?

B.
Take five.

  1. In constructing a life table what correction must be made for abnormal age and sex distribution of the population?
  2. Define the following terms: “Mortality,” “Expectation of Life,” “Mean Duration of Life.” How should you calculate the mean duration of life from the census returns for any community?
  3. How should you calculate the economic value of a population?
  4. What are some of the inaccuracies to which censes enumerations are liable?
  5. What is the nature of a statistical law? of what categories of social phenomena may statistical laws be formulated? in what sense are they laws? How do they bear upon freedom of the will in human conduct?
  6. How do the conditions of observation in social sciences differ from conditions of observation in the natural sciences?
  7. What do you understand by the law of criminal saturation?
  8. By what considerations should the Statistician be guided in in making selection of social phenomena for investigation?

Source: Harvard University Archives. Harvard University, Examination papers 1873-1915. Box 4, Bound volume: Examination Papers 1896-97. Papers Set for Final Examinations in Philosophy, History, Government, Economics, Fine Arts, Architecture, and Music in Harvard College, June 1896.

___________________________

1895-96.
Railway Transportation.

Course Enrollment for Economics 51.

For Graduates and Undergraduates:—

[Economics] 51. Professor Taussig. — Railway Transportation. — Lectures and written work. Hf. 3 hours. 1st half year.

Total 43: 6 Graduates, 27 Seniors, 7 Juniors, 3 Law.

Source: Harvard University. Report of the President of Harvard College, 1895-96, p. 64.

 

*  *  *  *  *  *  *  *  *  *

Final Examination

1895-96.
ECONOMICS 5.

  1. The means of transportation in the United States in 1855.
  2. Is there historical warrant for the assertion that in United States the construction and operation of railways have been left mainly to private enterprise?
  3. The resemblances and differences between the legislation of Iowa on maximum rates, and that of England.
  4. Are there good grounds for alarm at the tendency to consolidation and the growth of great systems among railways?
  5. “There was never a more mistaken idea than the idea that rates would be reduced if they were based on cost of service. The principle keeps rates up. If it is strictly applied, it makes it necessary that each item of business should pay its share of fixed charges.” Why? or why not?
  6. “It is not true that when the price falls below cost of production, people always find it for their interest to refuse to produce at a disadvantage. It very often involves worse loss to stop producing than to produce below cost.” Why and how, as to railways?
  7. The provisions of the Interstate Commerce Act which bear on

an agreement to maintain certain rates;
an agreement to divide earnings;
a lower rate for one hundred carloads than for one carload;
a postage-stamp rate;
a higher rate for a shorter than for a longer distance.

  1. Does the history of pooling arrangements in the United States justify the assertion that they tend to remove inequalities in the rates to shippers?
  2. The lessons of public railway management in Italy and in France.
  3. The evidence as to the financial and economic success of public railway management in Prussia.

Source: Harvard University Archives. Harvard University, Mid-year Examinations 1852-1943. Box 3, Bound volume Examination Papers, Mid-Years 1895-96.

Also found in: Harvard University Archives. Harvard University, Examination papers 1873-1915. Box 4, Bound volume: Examination Papers 1896-97. Papers Set for Final Examinations in Philosophy, History, Government, Economics, Fine Arts, Architecture, and Music in Harvard College, June 1896.

___________________________

1895-96.
History of Tariff Legislation in the United States.

Course Enrollment for Economics 61.

For Graduates and Undergraduates:—

[Economics] 61. Professor Taussig. — History of Tariff Legislation in the United States. Hf. 2 hours. 1st half year.

Total 88: 11 Graduates, 40 Seniors, 20 Juniors, 5 Sophomores, 12 Others.

Source: Harvard University. Report of the President of Harvard College, 1895-96, p. 64.

*  *  *  *  *  *  *  *  *  *

Final Examination

1895-96.
ECONOMICS 6.

Arrange your answers strictly in the order of the questions. One question, and one only, may be omitted.]

  1. What earlier legislation affected the provisions of the tariff act of 1789? What light does the earlier legislation throw on the character of this act?
  2. Was the argument for protection to young industries more applicable to cotton goods in 1816 than to silk goods in 1870?
  3. What changes were made, in 1833, in the duties on woollens, cottons, linens, and worsteds? Why the differences in policy?
  4. What were the grounds on which it was maintained, in 1828-32, that a tax on imports was virtually a tax on exports? How far was the assertion true?
  5. Mention points of similarity and points of difference between Webster’s speech of 1824 and Gallatin’s memorial of 1831.
  6. Should you say that the position of the protective system in public opinion was the same in 1870-90 as in 1816-32?
  7. Explain the legislation in regard to the duties on sugar in the acts of 1890 and 1894. Was the at of 1894 more advantageous than its predecessor to the planters? to the refiners? to the public?
  8. What do you believe would now be the effect, on domestic industries, of the free admission of (1) pig iron, (2) woollen goods, (3) linens?
  9. In what mode were the tea and coffee duties dealt with in the period 1840-60? in the period 1865-95? What explanation of the general course of policy can you give in either case?
  10. In what cases, if in any, are duties on imports a charge on the foreign producer?
  11. The significance of the events of 1860 for the tariff history of France and of England.
  12. Is there ground for saying that the drift since 1870 toward protective duties, in the United States and on the Continent of Europe, rests on the same general causes?

Source: Harvard University Archives. Harvard University, Mid-year Examinations 1852-1943. Box 3, Bound volume Examination Papers, Mid-Years 1895-96.

Also found in: Harvard University Archives. Harvard University, Examination papers 1873-1915. Box 4, Bound volume: Examination Papers 1896-97. Papers Set for Final Examinations in Philosophy, History, Government, Economics, Fine Arts, Architecture, and Music in Harvard College, June 1896.

___________________________

1895-96.
Financial Administration and Public Debts.

Course Enrollment for Economics 71.

For Graduates and Undergraduates:—

[Economics] 71. Dr. John Cummings. — Financial Administration and Public Debts. Hf. 3 hours. 1st half year.

Total 27: 1 Graduate, 8 Seniors, 12 Juniors, 1 Sophomore, 5 Others.

Source: Harvard University. Report of the President of Harvard College, 1895-96, p. 64.

*  *  *  *  *  *  *  *  *  *

Final Examination

1895-96.
ECONOMICS 7.

Divide your time equally between A and B. I and II may be treated as one question.

A.

  1. Give an account of the sinking fund provisions enacted by Congress 1790-1820; and of the management and refunding of the debt during this period.
  2. Examine and criticise the following account of the evolution of public credit, with a view to determining whether a government is ever justified in pledging the State to any definite policy of debt payment:—

“In this evolution, as in all others, there are transition stages: we have debts of long term, but secured by the pledging of public property or of income from taxes. Then we have a long period of redemption without such a pledge. The plan of discharging the debt simply on the ground of financial expediency, to which the debtor state has accustomed itself, presently takes the place of redemption simply at the instance of impatient creditors. Finally the question of redemption comes by mutual consent to be left entirely undetermined.”

B.

(Take any five of the questions following.)

  1. What effect upon the present worth of a security has lengthening the term for which it is to run?
  2. Give an account of the payment of the war indemnity to Germany.
  3. Discuss the “use and disuse of ‘relishes,’ gambling risks which are added in order to commend a public loan to the taste of creditors,” as a feature in the development of public credit.
  4. Compare the development of public credit in Prussia with that of Great Britain, at the beginning of this century.
  5. Examine and criticise the following selection:—

“As regards the relation of public control to the public credit, there is obviously a lone step taken in advance when the public control comes to he so employed as to not discriminate in its own favor.”

  1. [sic] Define the following terms, and illustrate: “budget,” “conversion,” “rolling annuity.”
  1. [sic] What influence has our Secretary of the Treasury over financial legislation, as compared with the influence of the English Chancellor of the Exchequer? Compare the manner of making up the estimates of public income and expenditure in England and in the United States; of appropriating funds out of the Treasury.

Source: Harvard University Archives. Harvard University, Mid-year Examinations 1852-1943. Box 3, Bound volume Examination Papers, Mid-Years 1895-96.

Also found in: Harvard University Archives. Harvard University, Examination papers 1873-1915. Box 4, Bound volume: Examination Papers 1896-97. Papers Set for Final Examinations in Philosophy, History, Government, Economics, Fine Arts, Architecture, and Music in Harvard College, June 1896.

___________________________

1895-96.
History of Financial Legislation in the United States.

Course Enrollment for Economics 82.

For Graduates and Undergraduates:—

[Economics] 82. Dr. J.A. Hill. — History of Financial Legislation in the United States. Hf. 2 hours. 2d half year.

Total 64: 5 Graduates, 22 Seniors, 18 Juniors, 6 Sophomores, 13 Others.

Source: Harvard University. Report of the President of Harvard College, 1895-96, p. 64.

 

*  *  *  *  *  *  *  *  *  *

Final Examination

1895-96.
ECONOMICS 8.

(N. B. — Omit one question under each of the five main divisions of the paper.)

I.

  1. Is the Independent Treasury System preferable to the use of banks as public depositories? Present the arguments on each side of the question, using any illustrations from the History of the United States that may occur to you.
  2. What illustrations does our financial history afford of difficulties that may arise from an exclusive reliance upon import duties as a source of revenue?

II.

  1. Describe the scheme which was adopted in 1790 for settling the accounts between the United States and the individual States. How did the assumption of the State debts affect the account?
  2. In what respects did the financial policy which the country pursued during the War of 1812 deviate from that which Gallatin had advocated in anticipation of war?
  3. What descriptions of treasury notes were issued during the War of 1812, and how did the successive issues indicate that the country was drifting towards a government paper currency?

III.

  1. The following extract is from a speech which Webster delivered in Congress on Jan. 2, 1815. The bank bill to which it refers was substantially the same as Dallas’ plan for a bank:—
    “What sort of an institution, sir, is this? It looks less like a bank than a department of the Government. It will be properly the paper money department. Its capital is Government debts; the amount of its issues will depend on Government necessities; Government, in effect, absolves itself from its own debts to the bank, and by way of compensation absolves the bank from its own contracts with others.”
    What features of the proposed bank did Webster refer to in his criticisms? What sort of a bank did he favor? What was the outcome of the movement for a bank at this session of Congress?
  2. What causes produced the surplus of 1836? When was there a somewhat similar situation in the later history of the country?
  3. State briefly where the public moneys of the United States have been kept at different periods since 1789.

IV.

  1. How did Secretary Chase execute the authority conferred upon him by the loan Acts of July 17 and Aug. 5, 1861, and in what respect was the course which he pursued open to criticism?
  2. The Legal Tender Act of March 3, 1863, contains the following clauses:—

And so much of the Act to authorize the issue of United States notes, and for other purposes, approved Feb. 25, 1862, and of the act to authorize an additional issue of United States notes, and for other purposes, approved July 11, 1862, as restricts the negotiation of bonds to market value, is hereby repealed. And the holders of United States notes, issued under and by virtue of said acts, shall present the same for the purpose of exchanging the same for bonds, as therein provided, on or before the first day of July, 1863, and thereafter the right so to exchange the same shall cease and determine.
Explain the meaning, object and effect of these provisions.

  1. How much assistance did the Government derive from the Direct Tax during the Civil War? Why is it probable that this form of taxation will never be resorted to again?

V.

  1. Give the main provisions of the Resumption Act of 1875? Why was it doubtful whether this Act would actually secure the resumption of special payments?
  2. State in general terms the changes effected in the form of the national debt (1) while McCulloch was Secretary of the Treasury, (2) under the Refunding Act of 1870, (3) by Secretary Windom in 1881.
  3. Give an account of the discussion which arose in 1867-68 on the question of paying the principal of the War debt in legal tender notes.

Source: Harvard University Archives. Harvard University, Examination papers 1873-1915. Box 4, Bound volume: Examination Papers 1896-97. Papers Set for Final Examinations in Philosophy, History, Government, Economics, Fine Arts, Architecture, and Music in Harvard College, June 1896.

___________________________

1895-96.
The Social and Economic Condition of Workingmen in the United States and in other countries.

Course Enrollment for Economics 9.

For Graduates and Undergraduates:—

[Economics] 9. Asst. Professor Edward Cummings. — The Social and Economic Condition of Workingmen in the United States and in other countries. 3 hours.

Total 67: 4 Graduates, 25 Seniors, 27 Juniors, 6 Sophomores, 5 Others.

Source: Harvard University. Report of the President of Harvard College, 1895-96, p. 64.

*  *  *  *  *  *  *  *  *  *

Mid-Year Examination.

1895-96.
ECONOMICS 9

(Arrange your answers in the order in which the questions stand. So far as possible illustrate your discussions by a comparison of different countries. Omit one question.)

  1. Contrast the structure of industry before machinery with the structure of modern industry.
  2. In what sense can there be said to be a law of invention? and how is this illustrated historically by the appearance and sequence of the great industrial inventions?
  3. How does machinery affect the demand for labor? the quality of labor? the family of the laborer? his real wage?
  4. Trade unionism vs. trades unionism; the old unionism vs. the new unionism. Explain the differences, and show how and when these phases have from time to time recurred during this century.
  5. How is Chartism related to other phases of the labor movement in England?
  6. The merits and the demerits of such trade-union organizations as you have thus far become acquainted with.
  7. Arbitration and Conciliation: (a) In what industries and in what forms have they succeeded best? (b) The present status and the prospects of industrial arbitration in England and in the United States.
  8. Taking the ordinary factory, how far is it possible or impossible to devise a system of remuneration which reconciles the interests of (a) workmen, (b) foremen, (c) employers, and (d) consumers? Explain carefully the merits and defects of the methods you propose to adopt or reject.
  9. In what respects does labor differ from other commodities? What ethical and economic consequences flow from these differences?
  10. How far, from time to time, has economic theory — Smith, Ricardo, Malthus, Mill, etc., — seemed to justify, and how far to suggest remedies for the industrial evils affecting wage-earners?
  11. The relation of cooperation to trade-unions, to profit-sharing, to socialism.

Source: Harvard University Archives. Harvard University, Mid-year Examinations 1852-1943. Box 3, Bound volume Examination Papers, Mid-Years 1895-96.

*  *  *  *  *  *  *  *  *  *

Final Examination

1895-96.
ECONOMICS 9.

(Arrange your answers in the order in which the questions stand. So far as possible illustrate your discussions by a comparison of different countries. Take the first six questions and one other.)

  1. Describe carefully the German system of compulsory insurance:

(a) To whom and to what proportion of the population it applies;
(b) The method of organization and of assessment in each case;
(c) The relation of the system to employer’s liability, to poor laws, friendly societies, etc.
(d) Arguments for and against the system.

  1. How far and with what modifications have such schemes been adopted or seriously proposed elsewhere?

(a) Contrast the plan in each case with the German plan;
(b) What circumstances seem to you to favor and what to hinder such action by the government?

  1. How far have voluntary organizations solved or failed to solve the problem of workingmen’s insurance, (a) in England? (b) in the United States?
  2. What light does the experience of France and of England during this century throw upon the good or the bad effect of attempts on the part of the government either to repress or to foster, (a) labor organizations; (b) coöperation; (c) friendly societies?
  3. In what other countries have you found instructive examples of such interference?
  4. Compare the experience and the legislation of the United States in regard to immigration, with the experience and legislation of other countries in which immigration problems have arisen.
  5. In what countries and in what ways have labor organization tended to drift into politics, and seek political remedies for industrial evils?

(a) Compare the experience of France, Belgium, Germany and English-speaking countries.
(b) What conclusion do you draw from such experience?

  1. What evidence do statistics of family income and expenditure furnish (a) in regard to the social condition of labor in staple industries of the United States and of competing countries? (b) in regard to cost of labor?
  2. What attempts have been made to perpetuate or reestablish certain aspects of the guild organization in European countries?
  3. Discuss the schemes adopted by governments, municipalities, etc., for meeting the “out-of-work problem.”
    What is the origin of that problem in the United States?

Source: Harvard University Archives. Harvard University, Examination papers 1873-1915. Box 4, Bound volume: Examination Papers 1896-97. Papers Set for Final Examinations in Philosophy, History, Government, Economics, Fine Arts, Architecture, and Music in Harvard College, June 1896.

___________________________

1895-96.
The Mediaeval Economic History of Europe.

Course Enrollment for Economics 10.

For Graduates and Undergraduates:—

[Economics] 10. Professor Ashley. — The Mediaeval Economic History of Europe. 2 hours.

Total 14: 7 Graduates, 5 Seniors, 2 Juniors.

Source: Harvard University. Report of the President of Harvard College, 1895-96, p. 63.

*  *  *  *  *  *  *  *  *  *

Mid-Year Examination.

1895-96.
ECONOMICS 10.

I.

To be first attempted by all.

Translate, and comment, on the following passages:

  1. Totius terrae descriptio diligens facta est, tam in nemoribus quam in pascuis et pratis, nec non in agriculturis, et verbis communibus annotata in librum redacta est.
  2. In Tineguella . . . sunt iiii hidae et dimidia ad geldum Regis. Et de istis tenent xx homines xx virgas terrae. Et xiii homines tenent vi virgas et dimidiam.
  3. Sicut traditum habemus a patribus, in primitivo regni statu post conquisitionem, regibus de fundis suis non auri vel argenti pondera sed sola victualia solvebantur.
  4. Plerique, cum aut aere alieno aut magnitudine tributorum aut injuria potentiorum premuntur, sese in servitutem dicunt nobilibus, quibus in hos eadem omnia sunt jura quae dominis in servos.
  5. Ceteris servis non in nostrum morem, descriptis per familiam ministeriis, utuntur. Suam quisque sedem, suos penates regit.

II.

Write on four only of the following subjects.

  1. The importance of the yardland in the rural economy of the Middle Ages.
  2. A history of the mark theory, from its first promulgation to its general acceptance.
  3. A comparison of the life of a medieval English village with that of a New England village of today.
  4. The Roman colonate.
  5. An account and criticism of Mr. Seebohm’s “Tribal System in Wales.”

Source: Harvard University Archives. Harvard University, Mid-year Examinations 1852-1943. Box 3, Bound volume Examination Papers, Mid-Years 1895-96.

*  *  *  *  *  *  *  *  *  *

Final Examination

1895-96.
ECONOMICS 10.

I.

To be first attempted by all.

Comment on the following passages, and translate those in Latin and French:—

  1. If a man agree for a yard of land, or more, at a fixed rent, and plough it; if the lord desire to raise the land to him to service and to rent, he need not take it upon him, if the lord do not give him a dwelling.
  2. Ego Eadward . . . rex . . . dedi X manentes in illo loco qui dicitur aet Stoce be Hysseburnam, cum omnibus hominibus qui in illa terra erant qando Ælfred rex viam universeæ carnis adiit.
  3. Magnates regni et alii minores domini qui tenentes habebant perdonarunt redditum de redditu ne tenentes abirent prae defectu servorum et caristia rerum.
  4. Whan Adam dalf and Eve span,
    Wo was thanne a gentilman?
  5. Nul ne deit rien achater a revendre en la vile meyme, fors yl sera Gildeyn.
  6. Cives Londoniae debent LX marcas pro Gilda telaria delenda ita ut de cetero non suscitetur.
  7. No one of the trade of Spurriers shall work longer than from the beginning of the day until curfew rings out at the church of St. Sepulchre.

II.

Write on four only of the following subjects:

  1. The economic and constitutional questions involved in recent discussions as to the beginnings of town life in mediaeval Europe.
  2. A comparison of a mediaeval merchant gild with a modern “trust,” and of a craft gild with a modern trade union.
  3. The extent and character of the public regulation of prices and wages in the later middle ages.
  4. The cause of the Peasant Revolt in 1381.
  5. The relation of the English Reformation to the origin of the Poor Laws.
  6. A criticism of Cunningham and McArthur’s Outlines of English Industrial History.

Source: Harvard University Archives. Harvard University, Examination papers 1873-1915. Box 4, Bound volume: Examination Papers 1896-97. Papers Set for Final Examinations in Philosophy, History, Government, Economics, Fine Arts, Architecture, and Music in Harvard College, June 1896.

___________________________

1895-96.
Banking and the History of the leading Banking Systems.

Course Enrollment for Economics 122.

For Graduates and Undergraduates:—

[Economics] 122. Professor Taussig. — Banking and the History of the leading Banking Systems. Hf. 3 hours. 2d half year.

Total 70: 10 Graduates, 30 Seniors, 19 Juniors, 4 Sophomores, 7 Others.

Source: Harvard University. Report of the President of Harvard College, 1895-96, p. 64.

*  *  *  *  *  *  *  *  *  *

Final Examination

1895-96.
ECONOMICS 12.

[Arrange your answers strictly in the order of the questions. Give some answer, however brief, to each question.]

  1. What was Bagehot’s opinion as to the advantage of a “many reserve” system as compared with a “single reserve” system? What light does American experience give?
  2. What important proposal made by Bagehot in Lombard Street has been adopted?
  3. What was the theory of the act of 1844? How far was that theory followed in the legislation on the Reichsbank of Germany?
  4. (a) Arrange in their proper order the following items, in which the figures stand for millions of marks.
Capital 150 Loans 800
Specie 800 Securities 50
Notes 1150 Other Assets 50
Deposits 350 Other Liabilities 50

(b) Consider what would be the significance of the statement if it were for the Reichsbank of Germany; assuming the limit of uncovered issue to be 300 millions of marks.

(c) Rearrange the items as they would appear if the statement were one of the condition of the Bank of England; assuming the limit of notes not required to be covered by specie to be 16 millions sterling = 400 million marks, and assuming that securities of any sort may be held against the uncovered issue. Consider then how the statement, thus rearranged, differs from a probable statement of the actual condition of the Bank of England in recent times.

  1. Does the Bank of France supply an elastic currency? Do the National Banks of the United States?
  2. “Redemption by the Treasury under the national bank legislation has been a convenient method of disposing of worn and soiled notes, and in case of accumulations of currency at special points has facilitated its rapid exchange for legal tender and specie. But nobody would say that this system has compelled any bank to face its notes in the same sense in which it has to face its liability for checks drawn against deposits.” Explain.
  3. Consider the effects on bank-note circulation and redemption of (1) exchange of notes among banks; (2) legislative prohibition of payment by a bank of notes other than its own; and give historical examples of the use of one or the other method.
  4. Does the United States Treasury now carry on a banking business? Did the Comptoir d’Escompte in 1848? The Prussian government in 1866?
  5. Does a banker lend his own money? the money of others?
  6. To what extent, and for what reasons, should the operations of savings-banks, private bankers, and trust companies, be excluded from consideration in this course?

Source: Harvard University Archives. Harvard University, Examination papers 1873-1915. Box 4, Bound volume: Examination Papers 1896-97. Papers Set for Final Examinations in Philosophy, History, Government, Economics, Fine Arts, Architecture, and Music in Harvard College, June 1896.

___________________________

1895-96.
Scope and Method in Economic Theory and Investigation.

Course Enrollment for Economics 132.

For Graduates and Undergraduates:—

[Economics] 132. Professor Taussig. — Scope and Method in Economic Theory and Investigation. Hf. 2 hours. 2d half-year.

Total 14: 11 Graduates, 3 Seniors.

Source: Harvard University. Report of the President of Harvard College, 1895-96, p. 63.

*  *  *  *  *  *  *  *  *  *

Final Examination

1895-96.
ECONOMICS 13.

  1. Compare Wagner’s enumeration of the problems within the scope of economic science with Keynes’s; and consider what doubts or objections there may be in regard to any of the problems mentioned by either writer.
  2. Explain and examine critically one of the following passages in Wagner:
    Section 63 (pp. 158-163).
    Section 70 (pp. 180-182).
  3. Illustrate the mode in which use is advantageously made of the deductive and the inductive method in regard to two of the following topics:

the causes which determine the general range of prices;
the prospects of socialism;
the prospects of coöperation.

  1. What peculiarities and difficulties appear for economic science if the choice of terminology and in definition? Illustrate.
  2. Is there ground for saying that the economic history of very recent times is of greater value for economic theory than the economic history of remote periods?
  3. What do you conceive to be the position in regard to method in economies of Ricardo? J.S. Mill? Roscher? Schmoller?

Source: Harvard University Archives. Harvard University, Examination papers 1873-1915. Box 4, Bound volume: Examination Papers 1896-97. Papers Set for Final Examinations in Philosophy, History, Government, Economics, Fine Arts, Architecture, and Music in Harvard College, June 1896.

___________________________

1895-96.
Communism and Socialism.

Course Enrollment for Economics 141.

For Graduates and Undergraduates:—

[Economics] 141. Asst. Professor Edward Cummings. — Communism and Socialism. — Utopias, ancient and modern. Hf. 2 hours. 1st half-year.

Total 15: 1 Graduate, 10 Seniors, 2 Juniors, 2 Sophomores.

Source: Harvard University. Report of the President of Harvard College, 1895-96, p. 63.

*  *  *  *  *  *  *  *  *  *

Final Examination

1895-96.
ECONOMICS 14.

(Arrange your answers in the order of the questions. Omit one.)

  1. The different senses in which the word Socialism is used. Where do you intend to draw the line between Socialism proper, and familiar forms of government interference and control — such as factory legislation, municipal water works, and government postal, telegraph or railroad services?
  2. “National communism has been confused with the common ownership of the family; tenure in common has been confused with ownership in common; agrarian communism with village commons.” Discuss the evidence.
  3. “Just as Plato had his Republic, Campanella his City of the Sun, and Sir Thomas More his Utopia, so Baboeuf had his Charter of Equality, Cabet his Icaria, St. Simon his Industrial System, and Fourier his ideal Phalanstery. . . . But the common criticism of Socialism has not yet noted the change, and continues to deal with the obsolete Utopias of the pre-evolutionary age.” What do you conceive to be the character of the change referred to? How far did the earlier Utopias anticipate the ideals of the modern social democracy?
  4. What indication of Socialistic tendency are to be found in the discipline of the Christian church? Explain the triple contract and its bearing on the doctrine of usury.
  5. “The Communistic scheme, instead of being peculiarly open to the objection drawn from danger of over-population, has the recommendation of tending in an especial degree to the prevention of that evil.” Explain Mill’s argument. Do you agree?
  6. To what extent are the theories of Karl Marx indebted to earlier writers in the 19th century?
  7. How far are the economic theories of (a) Lasalle, (b) Marx related to the theories of the so-called orthodox Economists? Explain critically.
  8. How far do you trace the influence of historical conditions in the social philosophies of Plato, More, Bacon, Rousseau, St. Simon, Karl Marx?
  9. What connection do you see between the teachings of Rousseau and (a) modern Socialism. (b) modern Anarchism?
  10. What, according to Hertzka, is the economic defect of the existing social and industrial system, and what is the remedy? Contrast “Freeland” with “Looking Backward.”

Source: Harvard University Archives. Harvard University, Mid-year Examinations 1852-1943. Box 3, Bound volume Examination Papers, Mid-Years 1895-96.

Also reprinted in. Harvard University, Examination papers 1873-1915. Box 4, Bound volume: Examination Papers 1896-97. Papers Set for Final Examinations in Philosophy, History, Government, Economics, Fine Arts, Architecture, and Music in Harvard College, June 1896.